Sie sind auf Seite 1von 111

B.Sc.

Physics Mechanics

B.Sc. Physics
Complete Notes of
MECHANICS

Chapter # 1: Vector Operations --------------Page 2


Chapter # 2: Particle Dynamics --------------Page 25
Chapter # 3: Special Theory of Relativity --------------Page 50
Chapter # 4: Work and Energy --------------Page 66
Chapter # 5: Conservation of Energy --------------Page 73
Chapter # 6: Collision --------------Page 84
Chapter # 7: Rotational Dynamics --------------Page 96

B.Sc. Physics Mechanics in Past Papers of University of Sargodha


B.Sc. Physics, Paper A, Annual 2017 --------------Page 105
B.Sc. Physics, Paper A, Annual 2016 --------------Page 106
B.Sc. Physics, Paper A, Annual 2015 --------------Page 107
B.Sc. Physics, Paper A, Annual 2014 --------------Page 108
B.Sc. Physics, Paper A, Annual 2013 --------------Page 109
B.Sc. Physics, Paper A, Annual 2012 --------------Page 110

OTHER UPDATED NOTES FOR STUDENTS OF B.Sc. PHYSICS ONLY


AVAILABLE AT www.HouseOfPhy.Blogspot.com
i. Waves and Oscillations
ii. Thermodynamics
iii. Electricity and Magnetism
iv. Electronics
v. Modern Physics

1
Contact Us: aliphy2008@gmail.com, www.facebook.com/HomeOfPhysics
B.Sc. Physics Mechanics
B.Sc. PHYSICS
VECTOR OPERATIONS

1. SCALARS
The physical quantities which are completely described by magnitude with proper unit are
called scalars. Mass, length, time density, energy, work, temperature and charge are the examples of
scalars.
Scalars can be added, multiplied and subtracted by ordinary rules of algebra.
2. VECTORS
The physical quantities which are completely described by magnitude, with proper unit and
direction are called vectors. Force velocity, acceleration, momentum, torque, electric field intensity
and magnetic field induction are the examples of vectors.
Vectors are added, multiplied and subtracted by vector algebra. However, parallel and
antiparallel vectors are added by ordinary algebra.
3. UNIT VECTOR
A vector having unit magnitude is called unit vector. It is used to describe the direction of any
vector. If we have a vector ⃗ , then a unit vector in the direction of ⃗ is written as:

̂
| |
Where ̂ is the unit vector in the direction of ⃗ and | | is its magnitude.
4. VECTOR ADDITION
The process in which two or more than two vectors are
added to get a single vector is called vector addition.
The vectors can be added graphically by head to tail rule.
According to this rule, the addition of two vectors ⃗ and ⃗⃗ consists
of following steps:
(i) Place the tail of vector ⃗ on the head of vector ⃗ .
(ii) Draw a vector from the tail of vector ⃗ to the head of vector ⃗ ,
called the resultant vector ⃗⃗ .

Important Note:
The vector sum ⃗ ⃗ and ⃗⃗ ⃗ has the same resultant ⃗⃗ ,
as shown in the figure. Therefore we can write:

2
Contact Us: aliphy2008@gmail.com, www.facebook.com/HomeOfPhysics
B.Sc. Physics Mechanics
⃗ ⃗ ⃗⃗ ⃗
So vector addition is commutative.
5. RESULTANT VECTOR
It is the sum of two or more vectors which along has the same effect as
the combined effect of all the vectors to be added.
The vector ⃗⃗ has the same effect as the combined effect of vectors
⃗ , ⃗⃗ , and ⃗⃗ . So ⃗⃗ is the resultant vector of the vectors ⃗ , ⃗ , and ⃗⃗ .

6. RECTANGULAR COMPONENTS

Consider a vector ⃗ represented by ̅̅̅̅ as shown in the figure. This vector can be
decomposed into three mutual perpendicular components
along x, y and z-axis. Let these components are denoted by
⃗ ,⃗ and ⃗ . These components form three sides of a
rectangular parallelepiped as shown.
Draw perpendicular ̅̅̅̅ on , then from figure:
̅̅̅̅ ̅̅̅̅ ̅̅̅̅

The vector ̅̅̅̅ is along z-axis and is denoted by ⃗ .


Therefore:
⃗ ̅̅̅̅ ⃗ ( )
Now draw perpendicular ̅̅̅̅ on x-axis, then:
̅̅̅̅ ̅̅̅̅ ̅̅̅̅
̅̅̅̅ is along x-axis and is denoted by ⃗ . ̅̅̅̅ is along y-axis and is denoted by ⃗ . Therefore,

̅̅̅̅ ⃗ ⃗

Putting values in (1), we have:


⃗ ⃗ ⃗ ⃗ ( )
If ̂ ̂ ̂ are the unit vectors along x, y and z-axis respectively, then:
⃗ ,̂ ⃗ ,̂ ⃗ ̂

Equation (2) can be written as:


⃗ ̂ ̂ ̂

The magnitude of this vector will be:

|⃗ | √

3
Contact Us: aliphy2008@gmail.com, www.facebook.com/HomeOfPhysics
B.Sc. Physics Mechanics
7. DIRECTION COSINES

Figure shows a vector ⃗ in space. Let this vector makes angles , , with x, y, z-axis respectively.
This
⃗ ̂ ̂ ̂

Now the unit vector in the direction of ⃗ is written as:



̂
| |
Where ̂ is the unit vector in the direction of ⃗ and | | is its magnitude.
̂ ̂ ̂
̂
| |

̂ ̂ ̂ ̂ ( )
| | | | | |

Now | |
,| |
,| |

Equation (1) becomes:


̂ ̂ ̂ ̂

Where are called direction cosines of vector ⃗ and are denoted as:

So the above equation becomes:


̂ ̂ ̂ ̂
Taking magnitude on both sides:

|̂| √


Squaring both side, we have:

So the sum of the squares of direction cosines is equal to unity.

Problem: Find the length (i.e., the magnitude) of the vector ⃗ ̂ ̂ ̂ . Also, calculate the
angles which this vector makes with the axes x, y and z.

Solution: The magnitude of vector ⃗ is described by formula:

|⃗ | √ ( )

Here Putting values in (1), we have:

√( ) ( ) ( ) √ √
Let ⃗ makes angles , , with x, y, z-axis respectively, then:

4
Contact Us: aliphy2008@gmail.com, www.facebook.com/HomeOfPhysics
B.Sc. Physics Mechanics

( ) ( )
| | | |

( ) ( )
| | | |

( ) ( )
| | | |

Problem: Find the resultant of the vectors ⃗ ̂ ̂ ̂ ⃗ ̂ ̂ ̂ ⃗ ̂ ̂ ̂.


Also, calculate the angles which resultant vector makes with the axes x, y and z.

Solution: If ⃗ the resultant of vectors ⃗ ⃗ ⃗ and are the rectangular components of


resultant, then:

The magnitude of resultant is given by:

|⃗ | √ √( ) ( ) ( )

√ √
Let ⃗ makes angles , , with x, y, z-axis respectively, then:

( ) ( )
| | | |

( ) ( )
| | | |

( ) ( )
| | | |
Problem: Find the angle between the direction of the vector given by the difference of the following
two vectors and the z-axis. ⃗ √ ̂ √ ̂ ̂ ⃗ √ ̂ √ ̂ ̂

Solution: Suppose the difference of the vectors: ⃗ ⃗ ⃗

√ √ √
√ √ √

The magnitude of resultant is given by:

|⃗ | √ √( √ ) ( √ ) ( ) √ √

Let ⃗ makes angle with z-axis, then:

( ) ( )
| | | |
5
Contact Us: aliphy2008@gmail.com, www.facebook.com/HomeOfPhysics
B.Sc. Physics Mechanics
8. SPHERICAL POLAR COORDINATES

Consider a vector ⃗ in three dimensions having components , , along x, y, z-axis


respectively.
Draw projection of ⃗ on xy-plane.
 The angle between ⃗ and z-axis is called polar angle.
 The angle between x-axis projection of ⃗ in xy-plane is
called Azimuthal angle.
From the figure, it is clear that:

Here ( ) are called spherical polar coordinates.


9. APPLICATIONS OF SPHERICAL POLAR COORDINATES
The spherical polar coordinates are superior than Cartesian Coordinates for the study of
Physical problems. For example, the gravitational force of the earth on distant objects has the
symmetry of sphere and its properties can be described in an easy way by the help of spherical polar
coordinates.
10. VECTOR DERIVATIVE
Consider a particle moving along a curve as shown. Its position at any time is given by a
vector ( ). With the passage of time, the direction and magnitude of this vector may change.
Let ( ) and ( ) denote the positions of the particle at time
and , respectively. Then, the displacement between two points
and is described as:
( ) ( )

is also a vector collinear with . As , point approaches

point and hord becomes the tangent at . When , the

ratio is written as and is called derivative of with respect to time

. But by definition is the velocity of the particle. Thus:

The acceleration being the rate of change of velocity is given by:

( )

6
Contact Us: aliphy2008@gmail.com, www.facebook.com/HomeOfPhysics
B.Sc. Physics Mechanics

In Cartesian coordinates: ̂ ̂ ̂
Now the position vector of the moving particle can also be written as:
( ) ( ) ̂( )
Then the derivative of ( ) is defined as:
( ) ( )

( ) ̂( ) ( ) ̂( )

̂
By Taylor’s Theorem: ( ) ( ) and ̂ ( ) ̂( )
̂
[ ( ) ] [ ̂( ) ] ( ) ̂( )

̂ ̂
( ) ̂( ) ( ) ̂( ) ( ) ( ) ̂( )

̂ ̂
[ ( ) ̂ ( )] ( )

̂ ̂
[ ( ) ̂ ( )] ( )

̂ ̂
[ ( ) ̂ ( )]

̂
( ) ̂( ) ( )

This is of the form:



( ⃗) ⃗ ⃗

̂
In equation (1), gives change in magnitude of and gives change in direction of

11. CIRCULAR MOTION


Consider an object is revolving along a circular path with constant angular velocity . The
position of the body revolving in a circle is given by:
( ) ̂( )
Suppose that the center of the circle is at origin O. Now the magnitude of remains constant
and the unit vector ̂ ( ) rotates at a constant rate. A circular motion is an example of a motion in two
dimension i.e., in a plane. So ̂ ( ) can be written as:
̂( ) ̂ ̂
7
Contact Us: aliphy2008@gmail.com, www.facebook.com/HomeOfPhysics
B.Sc. Physics Mechanics
̂( ) ̂ ̂ ( )
where is the angular velocity which is constant.
Velocity
The linear velocity of a body is then given as:

[ ̂ ( )]

̂( )
[ ̂ ̂]

[ ( )̂ ( ) ̂]
̂ ̂
Magnitude of velocity is given by:
| | √( ) ( )

√ ( )

√ ( )

Acceleration
The acceleration of the particle moving in a circle can be obtained by differentiating its
instantaneous velocity with respect to time:

[ ̂ ̂]

[ ̂ ̂]

[ ( )̂ ( ) ̂]
[ ̂ ̂]
̂( )
[ ̂ ( )]
( )
The magnitude of acceleration will be:
| | | |
| | | |
( )
The negative sign in equation (2) shows that acceleration is directed towards the origin i.e.,
towards the center of the circle and the equation (3) describes that the magnitude of acceleration is
proportional to the distance from the center (origin).

8
Contact Us: aliphy2008@gmail.com, www.facebook.com/HomeOfPhysics
B.Sc. Physics Mechanics
As, for the case of circular motion:

Equation (3) becomes:

( ) ( )

This is called centripetal acceleration.


12. SCALAR FIELD
Consider a scalar U e.g., temperature or density of a medium, which changes from point to point.
The region in which the scalar varies from point to point is called scalar field.
13. VECTOR FIELD

Consider a vector ⃗ e.g., electric field intensity which changes from point to point. The region in
which the vector varies from point to point is called vector field.
14. LINE INTEGRAL

Consider a vector ⃗ at point P of the curve AN of length l as shown in the figure. Take a small
element of length ‘⃗⃗⃗ ’, then the dot product of ⃗ and ⃗⃗⃗ is given by:
⃗ ⃗⃗⃗ |⃗ ||⃗⃗⃗ |
Or
⃗ ⃗⃗⃗ ⃗ ̂
Where ̂ is the unit vector in the direction of ⃗⃗⃗ .
Integrating the above expression over the entire length ‘l’, we have

∫ ⃗ ⃗⃗⃗ ∫

Or

∫ ⃗ ⃗⃗⃗ ∫⃗ ̂

This integral is called line integral of vector along the curve AB. The common example of line
integral is the definition of work.
15. SURFACE INTEGRAL

Consider a vector ⃗ at point P of a surface area . Take a small element


of the surface . Then
⃗ ⃗⃗⃗⃗ |⃗ ||⃗⃗⃗⃗ |

Where is the angle between ⃗ and outward drawn normal to . Or


⃗ ⃗⃗⃗⃗ ⃗ ̂
Where ̂ is the unit vector normal to surface .

9
Contact Us: aliphy2008@gmail.com, www.facebook.com/HomeOfPhysics
B.Sc. Physics Mechanics
Integrating the above expression over whole surface, we have

∫ ⃗ ⃗⃗⃗⃗ ∫

Or
∫ ⃗ ⃗⃗⃗⃗ ∫⃗ ̂

It is called the surface integral of vector ⃗ over the whole surface.


 If ⃗ is the velocity of fluid at any point, then ∫ ⃗ ⃗⃗⃗⃗ is called the rate of flow.

 If ⃗ is the electric field strength (or magnetic field strength) at any point, then ∫ ⃗ ⃗⃗⃗⃗ is gives
the electric flux (or magnetic flux).
16. VOLUME INTEGRAL
Consider a close surface having uniform volume charge density ( ). Take a small element of volume
having mass , then

Integrating over the whole volume, we get:

∫ ∫ ∫

And ∫ ∭ is called volume integral. Therefore,

∫ ∭

17. OPERATORS
These are the quantities whose operation on a function gives a new function. Operators
always operate on something placed after them. There are many types of operators e.g.,
 Number Operator ( )

 Differential Operator ( )

 Integral Operator (∫ ∫ )
 Logarithmic Operators ( )
But now we shall introduce a vector differential operator called del , denoted by ⃗ , which is
described as:

⃗ ̂ ̂ ̂

It can operate both on scalar and vector.

10
Contact Us: aliphy2008@gmail.com, www.facebook.com/HomeOfPhysics
B.Sc. Physics Mechanics
18. OPERATIONS OF ⃗
(i) Gradient Operation
(ii) Divergence Operation
(iii)Curl Operation
Gradient Operation

The operation of ⃗ on a scalar function is called gradient operation. If we have a scalar function U,
then
⃗ ⃗⃗⃗⃗⃗⃗⃗⃗⃗⃗

Note that ⃗⃗⃗⃗⃗⃗⃗⃗⃗⃗ is a vector. In Cartesian coordinates, ⃗ is written as:

⃗ ( ̂ ̂ ̂)

( ̂ ̂ ̂)

Divergence Operation

The operation of ⃗ on a vector function through dot product is called divergence operation. If we
have a vector function ⃗ , then:
⃗ ⃗ ⃗

Note that ⃗ is a scalar.

⃗ ⃗ ( ̂ ̂ ̂) ( ̂ ̂ ̂)

⃗ ⃗

Curl Operation

The operation of ⃗ on a vector function through cross product is called curl operation. If we have a
vector function ⃗ , then:
⃗ ⃗ ⃗

Note that ⃗ is a vector. In Cartesian Coordinates, ⃗ is described as:

̂ ̂ ̂
⃗ ⃗ || || ( ) ̂ ( ) ̂ ( ) ̂

19. GRADIENT OF SCALAR


The maximum rate of increase of a scalar function with respect to space in a particular
direction is called gradient of a scalar function.

11
Contact Us: aliphy2008@gmail.com, www.facebook.com/HomeOfPhysics
B.Sc. Physics Mechanics
20. LEVEL SURFACES (DEFINITION):
The surface in which all the point of a scalar field has same value is called level surface. An entire
scalar field can mapped out by level surfaces, each associated with a constant value of a scalar
function.

21. Show that ⃗⃗⃗⃗⃗⃗⃗⃗⃗⃗⃗ ⃗ , where U is a scalar point function.

Consider a scalar U e.g., temperature or density of a medium which changes from point to
point. The region in which the scalar varies from point to point is called scalar field. In such a field,
U has definite value at each point.
Let two level surfaces and associated with constant values and of the scalar
function, respectively.
Consider a point A on the surface with position vector ⃗⃗⃗⃗⃗ with respect to origin O.
Let ⃗⃗⃗⃗⃗ be the position vectors of any point B taken on surface as shown in the figure.
By head to tail rule, we have:
⃗⃗⃗⃗⃗ ⃗⃗⃗⃗⃗ ⃗⃗⃗⃗⃗
⃗⃗⃗⃗⃗ ⃗⃗⃗⃗⃗ ⃗⃗⃗⃗⃗
⃗⃗⃗⃗⃗
The shortest distance between the level surfaces
and is taken along the outward drawn normal ⃗⃗⃗⃗⃗
specified by the unit normal vector ̂. We can follow
different paths lengths between the level surfaces and . Let us consider on the path lengths ⃗⃗⃗⃗⃗
and ⃗⃗⃗⃗⃗ , where ⃗⃗⃗⃗⃗ is the shortest path length.

Now ⃗⃗⃗⃗⃗

and ⃗⃗⃗⃗⃗

We have:

Thus the rate of variation of takes place in the direction of directed normal ̂ and is called gradient
of U:

⃗⃗⃗⃗⃗⃗⃗⃗⃗⃗ ̂

Taking dot product of on both sides, we have:

⃗⃗⃗⃗⃗⃗⃗⃗⃗⃗ ̂

12
Contact Us: aliphy2008@gmail.com, www.facebook.com/HomeOfPhysics
B.Sc. Physics Mechanics
⃗⃗⃗⃗⃗⃗⃗⃗⃗⃗

Since U is the function of one variable, so we can replace partial differential coefficient by ordinary
differential coefficient. Therefore,

⃗⃗⃗⃗⃗⃗⃗⃗⃗⃗ ( )

In this scalar field U has a definite U is value at every point. So U is a function of


coordinates of point ( ) i.e.,
( )
Now, let the two points A and B has the coordinates ( ) and ( ) in the
scalar field. The displacement between P and Q is described as:
̂ ̂ ̂
The change in as we move from A to B (Total Differential ) is described (by Calculus) as:

( ̂ ̂ ̂) ( ̂ ̂ ̂) ( )

( ̂ ̂ ̂) ( ̂ ̂ ̂) ⃗

So, equation (2) becomes:


⃗ ( )
Comparing equation (1) and (3), we have:
⃗⃗⃗⃗⃗⃗⃗⃗⃗⃗ ⃗
Hence Proved.

Problem: Show that , where is the P.E. of the body.


Solution: As we know that for a scalar V,
⃗⃗⃗⃗⃗⃗⃗⃗⃗⃗ ( )
As the body moves from high P.E. to low P.E., the change in P.E. is equal to – . So,

( )
Comparing equation (1) and (2), we get:
⃗⃗⃗⃗⃗⃗⃗⃗⃗⃗
⃗⃗⃗⃗⃗⃗⃗⃗⃗⃗
Hence Proved
13
Contact Us: aliphy2008@gmail.com, www.facebook.com/HomeOfPhysics
B.Sc. Physics Mechanics
Problem: The P.E. of a body of mass m held at a height h above the surface of the earth is mgh. Find
the force of gravity (or weight) of the body.
Solution:
Given ( )
As ⃗⃗⃗⃗⃗⃗⃗⃗⃗⃗

( ̂ ̂ ̂)

( ̂ ̂ ̂) ( )

Put in equation (1), we have:


Thus equation (2) becomes:

( ( )̂ ( )̂ ( )̂)

( ( )̂ ( )̂ ( ) ̂)

( ̂ )

̂
This expression shows that force of gravity acts in the downward direction and the magnitude of
force of gravity is mg.
Problem: Show that if a vector is the gradient of a scalar function, then its line integral around a
closed path is zero.

Solution: Let be a vector which is equal to gradient of a scalar function


i.e.,
⃗⃗⃗⃗⃗⃗⃗⃗⃗⃗
Taking dot product of on both sides:
⃗⃗⃗⃗⃗⃗⃗⃗⃗⃗
Taking line integral from point A to point B on both sides:

∫ ∫ ⃗⃗⃗⃗⃗⃗⃗⃗⃗⃗ ( )

As

⃗⃗⃗⃗⃗⃗⃗⃗⃗⃗ ( ̂ ̂ ̂) ( ̂ ̂ ̂)

⃗⃗⃗⃗⃗⃗⃗⃗⃗⃗

14
Contact Us: aliphy2008@gmail.com, www.facebook.com/HomeOfPhysics
B.Sc. Physics Mechanics
⃗⃗⃗⃗⃗⃗⃗⃗⃗⃗
Expression (1) becomes:

∫ ∫ | |

This shows that line integral depends upon the values of at point A and B. If we have a close path
then the line integral around the close path is given by:

∫ ∫ | |

Hence proved that if a vector is the gradient of a scalar function, then its line integral around a closed
path is zero.

Problem: If a scalar function , then show that ⃗⃗⃗⃗⃗⃗⃗⃗⃗⃗ ⃗⃗⃗⃗⃗⃗⃗⃗⃗⃗ ( ) ̂ ̂


̂

Solution: As

⃗⃗⃗⃗⃗⃗⃗⃗⃗⃗ ̂ ̂ ̂ ( )

Also ̂ ̂ ̂

| | √
( )

( ) ( )

From differentiating (partially) both sides of equation (2) w.r.t x, we have

( ) ( )

15
Contact Us: aliphy2008@gmail.com, www.facebook.com/HomeOfPhysics
B.Sc. Physics Mechanics
Equation (3) becomes:

( )

Putting values in equation (1), we have:


⃗⃗⃗⃗⃗⃗⃗⃗⃗⃗ ̂ ̂ ̂

⃗⃗⃗⃗⃗⃗⃗⃗⃗⃗ ( ̂ ̂ ̂) ( )

⃗⃗⃗⃗⃗⃗⃗⃗⃗⃗

Hence Proved.

Problem: Given that ⃗ ̂ ̂ ̂ and ⃗ ̂ ̂ ̂ . Find ⃗⃗⃗⃗⃗⃗⃗⃗⃗ (⃗ ⃗ ).

Solution: As ⃗ ⃗ ( ̂ ̂ ̂) ( ̂ ̂ ̂)

⃗ ⃗ ( )( ) ( )( ) ( )( )
⃗ ⃗

Now ⃗⃗⃗⃗⃗⃗⃗⃗⃗ (⃗ ⃗ ) ⃗ (⃗ ⃗ )

⃗⃗⃗⃗⃗⃗⃗⃗⃗ (⃗ ⃗ ) ( ̂ ̂ ̂ ) (⃗ ⃗ )

(⃗ ⃗ ) (⃗ ⃗ ) (⃗ ⃗ )
⃗⃗⃗⃗⃗⃗⃗⃗⃗ (⃗ ⃗ ) ̂ ̂ ̂

⃗⃗⃗⃗⃗⃗⃗⃗⃗ (⃗ ⃗ )

( ) ( )
̂ ̂

( )
̂

⃗⃗⃗⃗⃗⃗⃗⃗⃗ (⃗ ⃗ ) ( )̂ ( )̂ ( )̂

22. Flux of a Vector Field


Consider a vector ⃗ e.g. velocity of a fluid which changes from point to point. The region in
which the vector varies from point to point is called vector field. In such a field, ⃗ changes from
point to point. So ⃗ is a function of position coordinates x,y,z and is written as:
⃗ ⃗( )

16
Contact Us: aliphy2008@gmail.com, www.facebook.com/HomeOfPhysics
B.Sc. Physics Mechanics
If we consider an element of area ⃗ in the vector field, then the scalar product of ⃗ and ⃗ is called
flux of vector field. Thus:
⃗ ⃗

If is the angle between ⃗ ⃗ , then:

|⃗ || ⃗ |

If the plane of area ⃗ is perpendicular to ⃗ , then , so

23. Show that ⃗ ⃗ ⃗

Consider a parallelepiped having side in a vector field. Let ⃗


be the value of vector at the center of parallelepiped, where
⃗ ̂ ̂ ̂

We consider two faces 1 and 2 of parallelepiped perpendicular to the x-


axis each of area .

 The value of x-component of ⃗ at the center of face-2

This may be taken as the value for the whole face-2 because the face is
very small.

 The flux entering the parallelepiped through face-1 ( )

Similarly,

 The value of x-component of ⃗ at the center of face-2

 The flux leaving the parallelepiped through face-2 ( )

( ) ( ) ( )

( ) ( )

Similarly, the net outward flux through parallelepiped along y-axis and along z-axis is given as:

( )

( )

17
Contact Us: aliphy2008@gmail.com, www.facebook.com/HomeOfPhysics
B.Sc. Physics Mechanics
( )

( ) ( ) ( )

( )

( ) ( )

Here is the volume of parallelepiped.

( )
( ) ( )

Now by the definition, the net flux per unit volume is called divergence of vector ⃗ . It is denoted by
⃗ . Thus:

This can be written as:

⃗ ( ̂ ̂ ̂) ( ̂ ̂ ̂)

⃗ ⃗ ⃗
Note: If flux entering a volume element is equal to flux leaving the volume element, then

Problem: Given a position vector ̂ ̂ ̂ . Evaluate .

Solution: As ⃗

( ̂ ̂ ̂) ( ̂ ̂ ̂)

Problem: If ̂ ̂ ̂ . Find divergence at point ( )

Solution: As ⃗ ⃗ ⃗

⃗ ( ̂ ̂ ̂) ( ̂ ̂ ̂)

⃗ ( ) ( ) ( )


Now
18
Contact Us: aliphy2008@gmail.com, www.facebook.com/HomeOfPhysics
B.Sc. Physics Mechanics
⃗ ( ) ( ) ( ) ( ) ( )( )
⃗ ( )

24. Prove that ⃗ ⃗ ⃗

Consider a fluid flowing with constant velocity ⃗ at any point ( ) with as its
rectangular components.
 Initially, we consider flow of fluid in plane,
along y-axis as shown in the figure. The
successive layers will move alike. However, if
is a function of z, then different layers in the -
plane will slide over or move relative to one
another. The velocity of different layers of fluid
goes on increasing along z-axis. Let be the
velocity of central layer along y-axis, then the
( )
velocity of the layer just above it will be [ ] and velocity of layer just below it will
( ) ( )
be [ ]. The factor gives the rotation of the fluid in clockwise direction around

x-axis from z to y.
 Similarly, we consider the flow of fluid in yz-plane along z-axis as shown in the figure. The
velocity of different layers of fluid goes on increasing along y-axis. Let be the velocity of
( )
central layer along z-axis, then the velocity of the layer just above it will be [ ] and
( ) ( )
velocity of layer just below it will be [ ]. The factor gives the rotation of the

fluid in anticlockwise direction around x-axis from y to z.


Taking counter-clockwise direction positive, the net rotation about x-axis must be
proportional to

( ) ̂

Similarly, Net rotation around y-axis will be:

( ) ̂

And net rotation around z-axis is:

( ) ̂

Now the total rotation of the fluid gives the Curl of vector ⃗ . Hence,

19
Contact Us: aliphy2008@gmail.com, www.facebook.com/HomeOfPhysics
B.Sc. Physics Mechanics

⃗ ( ) ̂ ( ) ̂ ( ) ̂ ( )

As,

⃗ ( ̂ ̂ ̂) ⃗ ̂ ̂ ̂

̂ ̂ ̂
Now, ⃗ ⃗ | |

⃗ ⃗ ( ) ̂ ( ) ̂ ( ) ̂ ( )

Comparing (1) and (2), we have:


⃗ ⃗ ⃗
Hence proved

Problem: If ⃗ ̂ ̂ ̂ , find ⃗ at the point ( )

Solution: ̂ ̂ ̂

̂ ̂ ̂
⃗ ⃗ ⃗ || ||

̂| | ̂| | ̂| |

̂( ( ) ( )) ̂( ( ) ( )) ̂( ( ) ( ))

(̂ ) (̂ ) ̂( )
( )̂ ̂ ̂

20
Contact Us: aliphy2008@gmail.com, www.facebook.com/HomeOfPhysics
B.Sc. Physics Mechanics
25. GAUSS’S DIVERGENCE THEOREM
The surface normal integral of vector taken over a closed surface is equal to the volume integral of
the divergence of a vector over the volume enclosed by the surface. Mathematically, it is described
as:

∫ ⃗ ⃗⃗⃗⃗ ∫ ⃗

∫⃗ ̂ ∫ ⃗

Proof
Consider small volume element having volume enclosed in surface S. then by
definition:
⃗ ⃗ ⃗

⃗ ( ̂ ̂ ̂) ( ̂ ̂ ̂)

⃗ ( )

Where are the components of A along x, y, z-axis respectively. Multiplying both sides
by , we get:

⃗ ( )

Integrating both sides, we get:

∭ ⃗ ∭( )

∫ ⃗ ∭ ∭ ∭

∫ ⃗ ∫ ∬ ∫ ∬ ∫ ∬

Since are functions of only one variable, so we can change partial derivative in total
derivative:

∫ ⃗ ∫ ∬ ∫ ∬ ∫ ∬

∫ ⃗ ∬ ∬ ∬

21
Contact Us: aliphy2008@gmail.com, www.facebook.com/HomeOfPhysics
B.Sc. Physics Mechanics

∫ ⃗ ∬ ∬ ∬

∫ ⃗ ∬ ∬ ∬

∫ ⃗ ∬( )

∫ ⃗ ∬ ⃗ ⃗⃗⃗⃗

∫ ⃗ ∫ ⃗ ⃗⃗⃗⃗

This theorem enable us to transform a surface integral into volume integral and vice versa.
Problem. Evaluate ∫ ̂ Where S is the close surface.

Solution.
By Gauss’s Divergence theorem:

∫ ̂ ∫⃗

∫ ̂ ∫( ̂ ̂ ̂) ( ̂ ̂ ̂)

∫ ̂ ∫( )

∫( )

∫( ) ∫ ∫

STOKE’S THEOREM
The line integral of a vector function around the closed curve (boundary edge) of a surface is
equal to the surface normal integral of the curl of vector function over that surface.
If ⃗ is a vector function, then mathematically:

∮ ⃗ ⃗⃗⃗ ∫ ⃗ ⃗⃗⃗⃗

22
Contact Us: aliphy2008@gmail.com, www.facebook.com/HomeOfPhysics
B.Sc. Physics Mechanics
Proof
Consider a surface enclosed by a curve ABCD. We
divide it into large number of small meshes. Let the area of a
mesh be ⃗⃗⃗⃗ .
As is the line integral per unit area. So

[ ⃗⃗⃗⃗ ] ⃗ ⃗⃗⃗⃗

Suppose that we take the line integral of all the meshes within the curve ABCD. If we add all
these line integrals, the line integral along each side of the mesh where the line integral is taken
twice, will cancel away and we are left with line integral only along the curve ABCD.
Hence the line integral ∮ ⃗ ⃗⃗⃗ taken over the curve ABCD and the surface integral

∫ ⃗ ⃗⃗⃗⃗ taken over the surface ABCD are equal. Therefore,

∮ ⃗ ⃗⃗⃗ ∫ ⃗ ⃗⃗⃗⃗

This is called Stoke’s theorem. This theorem enables us to transform a line integral into
surface integral and vice versa.
Prove the following vector identities:
a)
b) ⃗

Proof:
a)

̂ ̂ ̂ ̂ ̂ ̂
| | | |
⃗ (⃗ )
| | | |

b) ⃗

23
Contact Us: aliphy2008@gmail.com, www.facebook.com/HomeOfPhysics
B.Sc. Physics Mechanics

| |
⃗ ⃗ ⃗
| |

Problem: Prove that



⃗ ⃗ ⃗ ⃗ (⃗ ⃗ ) (⃗ ⃗ )⃗
⃗( ⃗) ⃗

( ⃗) ⃗

24
Contact Us: aliphy2008@gmail.com, www.facebook.com/HomeOfPhysics
B.Sc. Physics Mechanics

PARTICLE DYNAMICS
In this chapter we continue with applications of Newton’s laws. We study frictional forces and their
consequences. We discuss non-constant forces and show how to solve the equations of motion for such forces.
Finally, we show how using non-inertial reference frame produces effects that can be analyzed by introducing
inertial forces or pseudo-forces that are not caused by specific objects in environment.
Types of Interaction (Forces)
To understand the cause of force, we must have a detailed microscopic understanding of the
interaction of objects with their environment. On the most fundamental level, nature appears to
operate through a small number of fundamental forces. Physicists have traditionally identified four
basic forces. There are four fundamental forces in nature.
Gravitational force: This force originates due to the presence of matter.
Electromagnetic force: This force includes basic electric and magnetic interactions and is responsible
for the binding of atoms and the structure of solids.
Weak Nuclear Force: This force causes certain radioactive decay processes and certain reactions
among the fundamental particles.
Strong Force: This force operates among the fundamental particles (protons and neutrons) and is
responsible for biding the nucleus together.
Relative Strengths Ratios of Forces
For the case of two protons, the forces have the following relative strengths:
Strong Force: Relative Strength = 1
Electromagnetic Force: Relative Strength = 10 -2
Weak Nuclear Force: Relative Strength = 10 -7
Gravitational Force: Relative Strength = 10 -38
It is clear that the gravitational force is very weak and has negligible effects.
Electroweak Force
In 1967, a theory was proposed according to which weak and electromagnetic forces could be
regarded as parts of a single force called electroweak force.
Grand Unification Theories
There are new theories proposed for the combination of strong and electroweak forces into a
single force into a single framework.
Theories of Everything
The theories, which are proposed of the unification of all the four fundamental forces, are
called theories of everything.
There are some other forces for which the electromagnetic force is the origin. For example
contact forces such as normal force, frictional force, viscous force, tensile force, elastic force and
many others. Microscopically, all these forces originate with the force between the atoms.
25
Contact Us: aliphy2008@gmail.com, www.facebook.com/HomeOfPhysics
B.Sc. Physics Mechanics
Frictional Forces
As one object slides against the other, a force is produced between the surfaces. This force
opposes the motion of the bodies, called the force of friction.
So, the frictional force opposes the relative motion. Even when there is no relative motion,
frictional forces may exist between the surfaces. In an automobile, about 20% of the engine power is
used to counteract the frictional forces. Friction also causes wear and seizing of moving parts,
therefore a lot of effort is made to reduce the friction. But on the other hand, friction is very
important in our daily life, because it brings every rotating shaft to a halt. Without friction, we cannot
walk, we cannot hold a pencil and could not write. Also, wheeled transportation is only possible due
to friction.
Explanation: For example if a block of mass m is projected with initial velocity along a horizontal
table, then it will finally come to rest. This is due to the fact that there is force of friction between the
block and the surface of table which produces a negative acceleration . In an inertial frame, second
law of motion always associates a force with accelerated objects. In the present case, the table exerts
a force of friction on the sliding block with average value of . This reduced the velocity of the
block to zero.
Force of Static Friction
The frictional forces acting between the surfaces at rest with respect to each other are called
forces of static friction.
Consider a block of mass m placed on a horizontal surface. The weight of block is mg, which
is balanced by the normal force ⃗⃗ (reaction of the horizontal surface) as shown in the figure.
Suppose a force F is applied on the resting block which is balanced by the equal and opposite
force of static friction ⃗⃗ . As F increases, the force of static friction
also increases. Until ⃗⃗ reaches a certain maximum value just before
sliding the block. This force of static friction depends upon
 The normal force ⃗⃗
 The nature of the surfaces in contact
⃗⃗ ⃗⃗

Or (⃗⃗ ) ⃗⃗

Here (⃗⃗ ) is the maximum value of the force of static friction, just

before the sliding or moving of the block, is called the coefficient of static friction. It depends
upon the nature of the surfaces in contact.

26
Contact Us: aliphy2008@gmail.com, www.facebook.com/HomeOfPhysics
B.Sc. Physics Mechanics
Force of Kinetic Friction

When the value of applied force ⃗ is greater than the maximum force of static friction
(⃗⃗ ) , then the block starts moving and has accelerated motion, i.e.,

⃗ (⃗⃗ )

Let the value of the applied force ⃗ is so adjusted that the block
moves with uniform velocity v. in this case a force of friction is
also present which is called force of kinetic friction ⃗⃗⃗ . This
force also depends upon:
 The normal force
 The nature of the surfaces sliding against each other
When the block is moving with uniform velocity:
⃗⃗⃗ ⃗⃗
Where is called the coefficient of kinetic friction. It should
be noted that
(⃗⃗ ) ⃗⃗⃗

Also,

27
Contact Us: aliphy2008@gmail.com, www.facebook.com/HomeOfPhysics
B.Sc. Physics Mechanics
The Microscopic Basis of Friction
On the atomic scale, even the most finely polished surface is far from plane. For example, a highly
polished steel surface has irregularities. The surface irregularities is several thousands atomic
diameters.
When the bodies are placed in contact, then the actual microscopic area of contact is much
less than the true area of the surface. In a particular case, these areas can easily be in the ratio
1:10000. The actual microscopic area of contact is proportional to the normal force, because the
contact points deform plastically under the great stresses that develop at these points. Therefore,
many contact points actually becomes cold welded together.
This phenomenon of “surface adhesion” occurs because
at the contact points, the molecules on the opposite side of the
surfaces are so close together that they exert strong intermolecular
forces on each other.
The coefficient of friction depends upon many variables such as:
 The nature of surface of materials
 Surface finish
 Surface films
 Temperature
In the absence of air, oxide films may form on the opposite surface, which reduce the friction.

Q. The coefficient of static friction between Teflon and scrambled eggs is about 0.04. What is
the smallest angle from the horizontal that will cause the eggs to slide across the bottom of a
Teflon coated skillet?
Given Data: Coefficient of Static Friction
To Determine: Angle
Calculations: As ( )
Q. A block is at rest on inclined plane making angle with horizontal. As the angle of incline is
raised, it is found that slipping just begins at an angle of . What is the coefficient of static
friction between block and incline?
Given Data: Angle at which the slipping begins
To Determine: Coefficient of static friction
Calculations: Resolving weigh mg into rectangular components along and
perpendicular to the plane, we have and as shown in the
figure.

28
Contact Us: aliphy2008@gmail.com, www.facebook.com/HomeOfPhysics
B.Sc. Physics Mechanics
Here . In limiting equilibrium, we have:
∑ ( )

∑ ( )
Dividing (1) and (2):
Q. A baseball player with mass 79 kg, sliding into a base is slowed by a force of 470 N. What is
the coefficient of kinetic friction between the player and the ground?
Given Data: Force of Kinetic Friction , Mass of object
To Determine: Coefficient of Kinetic Friction
Calculations: For motion on horizontal plane

As

Q. What is the greatest acceleration that can be generated by a runner if the coefficient of
static friction between shoes and road is 0.95?
Given Data: Coefficient of Static Friction
To Determine: Acceleration
Calculations: For motion on horizontal plane
For present case: Force of static friction Applied Force by Runner
As

The Dynamics of Uniform Circular Motion and the Centripetal Force


Consider of body of mass m, which is moving with uniform speed v along a circular path
of radius . as the direction of the body changes continuously, therefore, it has variable velocity and
it has some acceleration a, which is directed radially inward i.e., towards the center of the circle. This
is called centripetal acceleration and is given by:

Hence is a variable vector because even though its magnitude remains constant, its direction
changes continuously. The net force acting upon the body is called centripetal force, which is given
by the Newton’s second law of motion:

∑⃗

|∑ ⃗ |

29
Contact Us: aliphy2008@gmail.com, www.facebook.com/HomeOfPhysics
B.Sc. Physics Mechanics
The body moving in the circle is not in equilibrium state, because the net force acting the body is not
zero. This force which is responsible for uniform circular motion and is always directed towards the
center of the circle is called centripetal force.
Examples
 For a disc of mass m on the end of a string and is moving with constant speed v along circular
path of radius R in horizontal plane. In this case, the centripetal force applied on the disc is
the tension T in the string.
 As the moon is revolving around the earth, therefore, the centripetal force is the gravitational
pull of the earth.
 For the case of the electrons revolving around the nucleus, the centripetal force is provided by
the electrostatic force of attraction between electron and the nucleus.
Q. In the Bohr model of the hydrogen atom, the electron revolves in a circular orbit around the
nucleus. If the radius is and the electron makes , find the
(a) speed of electron (b) acceleration of electron (c) force acting on electron.
Given Data: Mass , Radius ,
Angular Velocity
To Determine: Speed Acceleration Force
Calculations: (a) For circular motion
( )
(b)

(c)
Q. During an Olympic bobsled run, a European team takes a turn of radius 25 ft at a speed of
60 mi/h. What acceleration do the riders experience (a) in (b) in units of g
Given Data: Radius , Velocity
To Determine: Acceleration (a) in (b) in units of g

Calculations: As , Also
( )
(a)

( ) ( )
(b)
( )

30
Contact Us: aliphy2008@gmail.com, www.facebook.com/HomeOfPhysics
B.Sc. Physics Mechanics

The Conical Pendulum


A small body suspended in the vertical plane at the end of a string and revolving in horizontal circle
is called Conical Pendulum.
Figure shows a small body of mass m revolving in a horizontal
circle of radius R with constant speed v at the end of a string of
length L. As the body swings around, the string sweeps over the
surface of an imaginary cone. This device is called conical
pendulum.
The string describes a right circular cone of semi-angle
θ. Let ⃗ is the tension in the string and the weight ⃗ of the
conical pendulum is acting vertically downward. The string makes
an angle θ with vertical. According to Newton’s second law of
motion, the net force ∑ ⃗ ⃗ acting on the conical pendulum
is:

∑⃗ ⃗ ⃗

⃗ ⃗ ⃗
The tension in the string can be resolved into two rectangular components:
i. The redial component directed towards the center of the circle

Where the minus sign with the radial component implies that radial direction has been taken positive
outward from the axis
ii. The vertical component directed upwards

As there is no vertical acceleration, therefore, vertical forces are balanced:

( )

The centripetal force acting on the conical pendulum is equal to

the radial component of tension:

( )
Dividing equation (1) and (2), we get:
31
Contact Us: aliphy2008@gmail.com, www.facebook.com/HomeOfPhysics
B.Sc. Physics Mechanics

( )


This gives the constant speed of the conical pendulum.
Period of Revolution for Conical Pendulum
As the object is moving with constant speed, therefore the period of revolution T of the body in
conical pendulum is given by:



From figure: . Therefore:

√ √ ( )

It follows that the period of revolution is independent of the mass m of the body.
Q. A conical pendulum is formed by attaching a 53 g pebble to a 1.4 m string. The pebble
swings around in a circle of radius 25 cm. (a) What is speed of pebble? (b) What is
acceleration? (c) What is tension in the string?
Given Data: Mass , Length of string , Radius
To Determine: Speed Acceleration Tension

Calculations: For conical pendulum ( )

(a) √ √ ( )
( )
(b)

(c)

32
Contact Us: aliphy2008@gmail.com, www.facebook.com/HomeOfPhysics
B.Sc. Physics Mechanics
The Rotor
Q. Determine the minimum speed of rotation necessary to prevent the object from falling
inside rotor.
Ans. The rotor is a cylindrical shell capable of rotation about its vertical
cylindrical axis. A person enters the rotor, closes the door and stands up
against the wall. The rotor starts rotating about vertical axis. When it got
sufficient speed (pre-determined) then the floor below the person is opened
downward. The person does not fall but remains pinned up against the wall of
the rotor.
The forces on the man standing in the rotor are:
 Its weigh mg acting vertically downward
 Force of static friction between the man and the rotor wall acting upward which prevent falling of
the man
 The normal reaction N of the wall of the rotor on the man.
If the person does not fall then there must be no acceleration along vertical direction i.e.,
. Then by 1st condition of equilibrium along vertical direction:

( )

The normal force N is directed from wall toward man. The magnitude of the normal force is same as
the centripetal force because man doesn’t fall through or lift off the side of cylindrical shell. It
provides the necessary centripetal force on man to move in a circle of radius R with tangential
velocity . Therefore,

( )

Comparing equation (1) and (2), we have:

33
Contact Us: aliphy2008@gmail.com, www.facebook.com/HomeOfPhysics
B.Sc. Physics Mechanics
This is the expression f velocity of the rotor, beyond which the person in the rotor does not fall and
remains pinned against the wall of the rotor. The corresponding expression for angular velocity of
rotor will be:

The Banked Curve


Consider a car moving on a level road around a curved path of
radius R with constant speed .
The forces acting on the car are:
 Its weight mg acting vertically downward on the road
 Normal force N acting vertically upward on the car by the road-bed
A horizontal force on the car which provides centripetal force
necessary to keep the car along the circular path. This force is supplied by the side-way frictional
forces exerted by the road on the tyres of the car.
However, the frictional forces are not large enough at all time to be relied upon and also
cause un-necessary wear. For safe turn around the curved path, the road-bed is banked. The raising
outer edge of the track with respect to the inner edge is called Banking.
When the road-bed or the railway track is banked at an angle , the normal force has
vertical and horizontal components and respectively. The vertical component
balances the weight of the car. That is:

∑ ( )

As there is no acceleration along vertical, so

( )
The horizontal component of normal force supplies the centripetal force.

( )

Dividing equation (1) and (2), we get:

( )

34
Contact Us: aliphy2008@gmail.com, www.facebook.com/HomeOfPhysics
B.Sc. Physics Mechanics

It follows that the angle of banking depends on the speed of car and the radius of curvature of the
path and not on the mass of car.
A 10.7 kN car travelling at 13.4 m/s attempts to round an un banked curve with a radius of 61
m. (a) What force of friction is required to keep the car on its circular path. (b) If the curve
were not banked, what would be the minimum coefficient of friction between the tyres and
road that would keep traffic from skidding at this speed?
Given Data: Weight , Velocity , Radius
To Determine: (a) (b)

Calculations: As
( )
(a)

(b) As

Q. The coefficient of static friction between the tyres of a car and dry road is 0.62. The mass of
the car is 1500 kg. What maximum braking force is obtainable (a) on level road (b) on an
downgrade.
Given Data: Coefficient of friction , Mass of car , Downgrade angle

To Determine: Braking force (a) on level road (b) on an downgrade


Calculations: (a)
( )
(b)

Equations of Motion under Constant Force


A constant force produces constant acceleration and the acceleration is described as the
derivative of velocity. If a and v is the acceleration and velocity of the moving object, then
acceleration of the object is described as:
dv
a
dt
Or dv  a dt
Integrating the above equation between the limits:
35
Contact Us: aliphy2008@gmail.com, www.facebook.com/HomeOfPhysics
B.Sc. Physics Mechanics
At t  0 , velocity = v0

At t  t , velocity = v
v t

 dv  a  dt
v0 0

As the acceleration is constant:

v v a t 0
v t
0

v  v0  at

v  v0  at
The time derivative of position vector x is equal to the velocity v , i.e.,
dx
v
dt
Or dx  v dt
Putting the value of v from equation (1), we have:
dx  (v0  at ) dt
Integrating the above equation between the limits:
At t  0 , Position w r t origin = x0

At t  t , velocity = v
x t

 dx   ((v
x0 0
0  at ) )dt

x t t

 dx   v0 dt   at dt
x0 0 0

t
t2
x x  v0 t 0  a
x t
0
2 0
2
t
x  x0  v0t  a
2
t2
x  x0  v0t  a
2
Example of Non-Constant Forces
There are some forces which are not constant but these forces change with respect to time,
velocity or position.
Forces Depending on Time
To stop a moving car, brakes are applied slowly at first and then more strongly as the car
slows. In this case, the braking force depends on the time during the interval when the car is slowing.

36
Contact Us: aliphy2008@gmail.com, www.facebook.com/HomeOfPhysics
B.Sc. Physics Mechanics
Another example of time dependent force is that force which is applied by the sound waves on the air
molecules during their propagation. As the sound waves vary sinusoidally with respect to time,
therefore, the forces also change sinusoidally with respect to time.
Force Depending on Velocity
When a body is moving through a fluid medium, such as air or water, then the frictional
force or drag force acting upon the body increases with increase in velocity of the body.
In case of free fall, the drag force increases up to the limit that it balances the weight of
the body and then the body falls with constant velocity, known as terminal velocity. To approach the
limit of terminal velocity, the free fall must be of the order of 100 m or so. Similarly the projectile
motion is also affected severely by the drag force due to which the range can be reduced on half or
less.
If we walk slowly in a swimming pool, we feel only a small resistive force. But it we try
to walk quickly, the resistive force will also increase.
Forces Depending upon the Position
The restoring force applied by a spring on a body of mass m is the example of the force which
depends on position. The resorting force F is directly proportional to the displacement x, of the body
from mean position:
F  x
F   kx
Using Newton’s 2nd law of motion:
ma   kx
Where m is mass of the object and a is the acceleration with which the object is moving.
k
a  x
m
The restoring force will be zero at mean position and it will become maximum at extreme position.
In a similar way, the acceleration also increases and decreases as the position of the vibrating body
changes.
Time Dependent Forces (Analytical Method)
Let a(t) is the time dependent acceleration due to a time dependent force. Then in is given by the
expression:
dv
a (t ) 
dt
Or dv  a(t ) dt
Integrating the above equation between the limits:

37
Contact Us: aliphy2008@gmail.com, www.facebook.com/HomeOfPhysics
B.Sc. Physics Mechanics
 At t  0 , velocity = v0

 At t  t , velocity = v
v t

 dv   a(t ) dt
v0 0

t
v v   a(t ) dt
v
0
0

t
v  v0   a(t ) dt
0

t
v  v0   a(t ) dt
0

Once we have v(t ) , we can calculate x(t )


The time derivative of position vector x is equal to the velocity v , i.e.,
dx(t )
v(t ) 
dt
Or dx  v(t ) dt
Integrating the above equation between the limits:
 At t  0 , Position w r t origin = x0

 At t  t , velocity = v
x t

 dx   v(t ) dt
x0 0

t
x x   v(t ) dt
x
0
0

t
x  x0   v(t ) dt
0

t
x  x0   v(t ) dt
0

Drag Forces and the Motion of Projectile


Raindrops fall from the clouds whose height h above the ground is about 2 km. the

expected velocity of the raindrop on striking the ground is v  2 gh  200 ms 1 . But the actual
velocity of the raindrop is much smaller. This is due to the drag force i.e., frictional force of air on
the raindrop.

38
Contact Us: aliphy2008@gmail.com, www.facebook.com/HomeOfPhysics
B.Sc. Physics Mechanics
The drag force acting on an object depends upon its velocity. Greater the velocity, greater
is the drag force. The velocity of the object can increase to a constant value which is known as
terminal speed. In this case, the force and acceleration is velocity dependent.
dv
a (v ) 
dt
dv
dt 
a (v )
Integrating the above equation between the limits:
 At t  0 , velocity = v0

 At t  t , velocity = v
t v
dv
 dt  
0 v0
a (v )
v
dv
t 
v0
a (v )

SAMPLE PROBLEM
Consider an object of mass m falling in air experiences a draf force D, which increases linearly with
velocity:
D v
D bv
Here b is the constant depending on the properties of the object (its size and shape) and also on the
properties of the fluid (especially its density). We have to find the velocity as a function of time, v(t )
, when the mass is dropped from rest v0  0 .

Solution
The net force acting on the object in the downward direction is

F y  mg  bv

ma  mg  bv
b
ag v ---------- (1)
m
For the case of a velocity dependent force, we can write:
dv
a (v ) 
dt
dv
dt 
a (v )
Integrating the above equation between the limits:
39
Contact Us: aliphy2008@gmail.com, www.facebook.com/HomeOfPhysics
B.Sc. Physics Mechanics
 At t  0 , velocity = v0

 At t  t , velocity = v
t v
dv
 dt   a(v)
0 v0

v
dv
t  ---------- (2)
v0
a (v )

Putting values from equation (1), we have:


v
dv
t 
v0  g 
b 
 v
 m 
For the present case, v0  0 , therefore
v
dv
t 
0 g 
b 
 v
 m 
v
dv
t  m
0 
mg  bv 

bdv
v
m
t  
b 0  mg  bv 

m
ln  mg  bv  0
v
t 
b
m
t  ln  mg  bv   ln(mg ) 
b
m  mg  bv 
t  ln  
b  mg 

 mg  bv  bt
ln  
 mg  m
 bt 
 mg  bv    m 
 e
 mg 
 bt 
 
mg  bv  mg e  m

 bt 
 
bv  mg  mg e m

   
 bt 

bv  mg 1  e m  
 
 

40
Contact Us: aliphy2008@gmail.com, www.facebook.com/HomeOfPhysics
B.Sc. Physics Mechanics
mg  
 bt 
 
v 1  e  m
 ---------- (3)
b  

Case 1. When t is small:
x2
When t is small, then by using formula e x  1  x   ..... for x  1 in equation (3), we
2!
have:
mg  bt 
v    gt
b m
It means that for short interval of time, the object is falling freely under the action of
gravity. The effect of drag force will be negligible for this case.
Case 2. When t is large:
When t is large, then
 bt 
 
e m
0
The equation (3) will become:
mg
v
b
This velocity is known as terminal velocity.
Exercise Problem 59: An object is dropped from from rest. Find the terminal speed assuming
that the drag force is given by D  bv 2
Solution. Assuming Newton’s 2nd law:
Fnet  mg  D  mg  bv 2

dv
m  mg  bv 2
dt
dv bv 2
g
dt m
dv
 dt
 bv 2 
g 
 m 

dv
 dt
b  mg 2
 v 
m b 

mg
 vT
b

41
Contact Us: aliphy2008@gmail.com, www.facebook.com/HomeOfPhysics
B.Sc. Physics Mechanics
dv b
 dt
 vT  v
2 2
m

Integrating both sides:


dv b
 v 2
 v 2

 m
dt
T

v
1 v v b t
ln T  t0
2vT vT  v 0 m

1   vT  v   vT  0   b
ln    ln    t
2vT   vT  v   vT  0   m

1   vT  v   b
ln    ln 1   t
2vT   vT  v   m
ln(1)  0

  vT  v   2vT b
ln    0  t
  vT  v   m

 v  v  2vT b v v  2v b 
ln  T  t T  exp  T t 
 vT  v  m vT  v  m 
Applying Componendo-Dividendo Rule:
 2v b   2v b   2v b 
exp  T t   1 exp  T t   1 exp  T t   1
vT  v  vT  v  m   2vT   m   vT   m 

vT  v  vT  v  2v b  2v  2v b  v  2v b 
exp  T t   1 exp  T t   1 exp  T t   1
 m   m   m 
 2v b 
exp  T t   1

v
  m 
vT  2v b 
exp  T t   1
 m 
  2vT b  
 exp  m t   1 
v  vT    
 exp  2vT b t   1 
  
 m  

  2vT b    2v b   
 exp  t  1  exp   T t   
 m   m  
v  vT 
  2vT b    2vT b   
 exp  m t  1  exp   m t   
     

42
Contact Us: aliphy2008@gmail.com, www.facebook.com/HomeOfPhysics
B.Sc. Physics Mechanics
  2vT b   
 1  exp   t  
   m  
v  vT
  2vT b   
 1  exp   m t   
   
Terminal Speed:
For terminal Speed, put t   :
 2v b 
When t   , exp   T t   0
 m 
1  0 
v  vT   vT
1  0 
Projectile Motion with Air Resistance
The two dimensional projectile motion is also affected by the drag force due to resistance of air. The
height as well as the range of projectile is reduced. For example if a base ball is projected with initial
velocity of 45 ms-1 at an angel of 600 with the horizontal, then its range is reduced from 179 m to 72
m and the maximum height is reduced from 78 m to 48 m. also the trajectory is no symmetric about
the maximum the descending motion is much steeper than ascending motion. The projectile strikes
the ground at an angel of - 790 . The drag force depends upon the velocity of projectile. If the wind
is blowing, the calculation must be changed accordingly and results will differ.
Projectile Motion
When a body is projected at an angel with the horizontal and it moves freely under the
action of gravity is called a projectile. Projectile motion is an example of two dimensional motion in
which the objects moves with constant acceleration.
Suppose a projectile of mass m is projected at an angle 0 with the horizontal with initial velocity v0

and it moves in xy-plane. Let r r is the position vector and v be its velocity at any time, then
according to second law of motion:
ma = F = - mg ˆj

d 2r dv
m 2 =m = - mg ˆj
dx dx
dv
 = - g ˆj
dx
Integrating both sides with respect to time, we get:
dv
 dx dt = -  g ˆj dt

v = -g ˆj t  A1

43
Contact Us: aliphy2008@gmail.com, www.facebook.com/HomeOfPhysics
B.Sc. Physics Mechanics
Initially, at t  0, v  v0 , so A1  v0

v = v0 -g ˆj t ---------- (1)
As the motion of the object is in two dimensions, so the equation (1) can be written in terms of
rectangular components as:

(vx ˆi + v yˆj) = (v0x ˆi + v0yˆj) -g ˆj t

Or v x ˆi + v y ˆj = v 0x ˆi +  v 0y - gt  ˆj

Comparing coefficients of î and ˆj on both sides:


vx = v0x Or vx = v0 cos 0

v y = v0y - gt Or v y = v 0 sin 0 - gt

The equation (1) can be written as:


dr
v= = v0 -g ˆj t
dt
Integrating both sides, we get:

 dt dt =   v 
dr
0 - g ˆj t dt

1
r = v0t - gt 2 ˆj  A2
2
Initially, at t  0, r  0 , so A2  0
Thus
1
r = v0t - gt 2 ˆj ---------- (2)
2
As the motion of the object is in two dimensions, so the equation (2) can be written in terms of
rectangular components as:

ˆ = (v ˆi + v ˆj)t - 1 gt 2 ˆj
(xiˆ + yj) 0x 0y
2
 1 
Or xiˆ + yjˆ = v0x t ˆi +  v0yt - gt 2  ˆj
 2 

Comparing coefficients of î and ˆj on both sides, we get:


x = v0x t or x = v0 cos 0t

1 2 1 2
y = v0yt - gt or y = v0 sin 0t - gt
2 2

Trajectory of the Projectile


44
Contact Us: aliphy2008@gmail.com, www.facebook.com/HomeOfPhysics
B.Sc. Physics Mechanics
x
As x = v0 cos 0t  t 
v0 cos 0
Putting this value of t in
1 2
y = v0 sin 0t - gt
2
2
 x  1  x 
y = v0 sin 0  - g 
 v0 cos 0  2  v0 cos 0 
1 x2
y = x tan  0 - g 2
2 v 0 cos 2  0
We can write above equation as:
y = a x - b x2 ---------- (3)
1 g
Where a  tan 0 , and b 
2 v0 cos 2 0
2

Equation (3) is the equation of parabola. So the trajectory of projectile is parabola.


Magnitude of Velocity at any Instant
The magnitude of velocity can be find out by using formula:

v  vx 2  vy 2

 v0 cos0    v0 sin 0 - gt 
2 2
v

v  v0 2 cos2 0  v0 2 sin 2 0 + g 2t 2  2v0 gt sin 2 0

 
v  v0 2 cos2 0  sin 2 0 + g 2t 2  2v0 gt sin 2 0

v  v0 2 + g 2t 2  2v0 gt sin 2 0

Direction of Velocity at any Instant


The angle θ which the velocity makes, at instant, with horizontal can be find out by using
expression:
Vy v0 sin  0 - gt
tan   
Vx v0 cos  0
Time to Reach at Maximum Height ( )
As the vertical component of velocity is
v y = v 0 sin  0 - gt

At highest point v y = 0 , therefore,

0= v0 sin 0 - gt

45
Contact Us: aliphy2008@gmail.com, www.facebook.com/HomeOfPhysics
B.Sc. Physics Mechanics
gt  v0 sin 0

v 0 sin  0
tm 
g
Time of Flight (T)
It is the time taken by the projectile from the point of projection to come back to the level of
projection.
1 2
y = v0 sin 0t - gt
2
As vertical displacement of the projectile y  0 , so
1 2
0 = v0 sin 0t - gt
2
1 2
gt = v0 sin 0t
2
2v0 sin 0
t=
g
Here t is the time of flight, i.e., t =T:
2v0 sin  0
T=
g
Horizontal Range
It is the horizontal distance covered by the projectile. As the horizontal component of velocity for a
projectile remains constant (ax  0) , so by using the 2nd equation of motion:

R  vx  T

 2v sin  0 
Where vx   v0 cos 0  is the horizontal component of velocity and T  = 0  is the time of
 g 
flight:
2v0 sin 0
R  v0 cos 0 
g

2v0 2 sin 0 cos 0


R
g

v0 2  2sin  0 cos  0 
R
g

v0 2 sin 20
R
g

46
Contact Us: aliphy2008@gmail.com, www.facebook.com/HomeOfPhysics
B.Sc. Physics Mechanics
Maximum Range
The range of the projectile will be maximum, when
sin 20  1

 20  sin 1 1  2 0  900

  0  450

Thus the projectile will have the maximum range when it will be projected at an angle of 450 .
Therefore:
v0 2
R
g
Non-Inertial Frames and Pseudo Forces
To apply the classical mechanics in non-inertial frames, we must introduce additional
forces known as pseudo-forces. Unlike other forces, we can not associate pseudo-forces with any
particular object in the environment of the body on which they act. Moreover, if we view the body
from an inertial frame, the pseudo forces disappear. Pseudo forces are simply devices that permit us
to apply classical mechanics in the normal way to events if we insist on viewing the events from a
non-inertial reference frame.
Linearly Accelerated References Frames
Consider an observer S' riding in a van that is moving at constant velocity. The van
contains a long air-track with a frictionless 0.25 glider resting at one end. The driver of the van
applies the brakes, ant the van begins to decelerate. An observer S on the ground measures the
constant acceleration of the van to be -2.8 ms-2. The observer S' riding in the van is therefore in a
non-inertial frame of reference when the van begins to decelerate. The observer S' observes the
glider to move down the track with an acceleration of 2.8 ms-2.
For ground observer S , who is an inertial frame of reference, the analysis is straight
forward. The glider, which had been moving forward at constant velocity before the van started to
brake, simply continues to do so. According to S , the glider has no acceleration and therefore no
horizontal force need be acting on it.
Observer S' , however, sees the glider accelerate and can find no object in the environment
of the glider that exerted a force on it to to provide its observed forward acceleration. To preserve the
applicability of Newton’s second law, S' must assume that a pseudo force acts on glider. According
to S' , this force F ' must equal ma ' , where a '( a) is the acceleration of the glider measured by
observer S' . The magnitude of this pseudo force is:
F '  ma '  (0.25 kg )(2.8 ms 2 )  0.70 N
And its direction is the same as a ' , that is towards the front of van.
47
Contact Us: aliphy2008@gmail.com, www.facebook.com/HomeOfPhysics
B.Sc. Physics Mechanics
This force which is very real from the point of view of S ' , is no apparent to the ground observer S .
Driving a Car on Circular Path
Pseudo forces are very real to those that experience them. Imagine yourself riding in a car that is
rounding a curve to the left. To a ground observer, the car is experiencing a centripetal acceleration
and therefore constitutes a non-inertial reference frame. To the ground observer, who is in inertial
frame of reference, this is quite natural: your body is simply trying to obey the Newton’s first law
and moves in a straight line. From your point of view in non-inertial frame of reference of car, you
must ascribe your sliding motion to a pseudo-force pulling you to the right. This type of pseudo force
is called centrifugal force meaning a force directed away from center.
Centrifuge Machine
Pseudo forces can be used as the basis of practical devices. Consider the centrifuge, one of the most
useful of laboratory instruments. As a mixture of substances moves rapidly in a circle, the more
mv 2
massive substances experience a larger centrifugal force and move further away from the axis
r
of rotation. The centrifuge, thus uses a pseudo-force to separate substance by mass, just as mass
spectrometer uses electromagnetic force to separate atoms by mass.
Limitations of Newton’s Laws
In 20th century, the physical world has experienced three revolutionary developments:
 Einstein’s Special Theory of Relativity (1905)
 Einstein’s General Theory of Relativity (1915)
 Quantum Mechanics (1925)
Special theory of relativity teaches that we can’t extrapolate the use of Newton’s laws to
particles moving at speed comparable to the speed of light. General theory of relativity shows that
we can’t use Newton’s laws in the vicinity of very large gravitational force. Quantum mechanics
teaches us that we can’t extrapolate the Newton’s laws to the objects as small as atom.
Special theory, which involves a distinctly non-Newtonian view of space and time, can be
applied under all circumstances, at high speed and low speeds. In the limit of low speed, it can be
shown that the dynamics of special reduces directly to the Newton’s laws.
Similarly, general theory can be applied to weak as well as strong gravitational fields, but
its equation reduces to Newton’s laws for weak forces.
Quantum mechanics can be applied to the individual atoms, where certain randomness in
behavior is predicted. To ordinary objects containing huge number of atoms, the randomness
averages out to give Newton’s laws once again.

48
Contact Us: aliphy2008@gmail.com, www.facebook.com/HomeOfPhysics
B.Sc. Physics Mechanics

Special Theory of Relativity

Frame of Reference
A coordinate system relative to which the measurements are taken is known as frame of
reference. A coordinate system in which the law of inertial is valid is known as inertial frame of
reference, while the accelerated frame is known as non-inertial frame of reference.
Special Theory of Relativity
Postulates of Special Theory of Relativity:
In 1905, Albert Einstein formulated his special theory of relativity in terms of two postulates:
1. Principle of Relativity
The laws of Physics have the same form in all frames of reference moving with constant
velocities with respect to one another. It can also be stated as
“The laws of Physics are invariant o a transformation between all inertial frames”
2. Principle of Constancy of Speed of Light
The speed of light in free space has the same value for all observers regardless of their state of
motion. It can also be stated as:
The speed of light in free space has the same value “c” in all inertial frames of references.
Relativistic Effects:
Theory of Relativity shows the effects of relative motion on physical quantities. These effects
are observed at relativistic speed. (The speed is called relativistic speed, where c is speed of

light.)
Discussion:
The first postulate shows that laws of Physics are absolute and universal and are same for all
inertial observers. So the laws of Physics that hold for one inertial observer can’t be violated for any
other inertial observer.
To understand 2nd postulate, consider three observers A, B and C at rest in three different
inertial frames.
 A flash of light emitted by observer A is observed by him to travel at speed c.
 If the frame of B is moving away from A at a speed of then according to Galilean

Kinematics, B measure the speed of flash emitted by A: .

 If the frame C is moving towards A with a speed of then according to Galilean

Transformation, C measures the value for the speed of flash emitted by A.

49
Contact Us: aliphy2008@gmail.com, www.facebook.com/HomeOfPhysics
B.Sc. Physics Mechanics
nd
However according to 2 postulate all the three observers measure the same speed of flash of
light. However ordinary objects don’t obey 2nd postulate e.g.,

( ) ( ) ( )

But velocities of waves and particles moving at speeds close to c behave in this wave. When
Einstein put forward these postulates, there was no experimental test for the verification of these
postulates. However, in 1964 a proton accelerator produced a beam of neutral pions ( ) which
rapidly decay into :

Now are electromagnetic waves and move with the speed of light. The speed of moving
Pions was measured equal to .
According to Galileo, the emitted in the direction of motion of Pions should have a speed
equal to But the measured speed of was equal to . This is consistent with
nd
2 postulate.
Galilean Transformation
Consider two observers in two different inertial frames and . Frame is at rest and is
moving with uniform velocity along x-axis with respect to frame . Suppose at , the origins
of two frames coincide.
Both the observers observe the same event. The
position and time of event observed by is denoted by
( ) and position and time of the event observed
by is denoted by ( ). According to Galilean
Transformation:

{ }

Fundamental Equation of Special Theory of Relativity


Consider two observers in two different inertial frames and .
Frame is at rest and is moving with uniform velocity along
x-axis with respect to frame . Suppose at , the origins of
two frames coincide.
Both the observers observe the same event. The position
and time of event observed by is denoted by ( ) and
position and time of the event observed by is denoted by
50
Contact Us: aliphy2008@gmail.com, www.facebook.com/HomeOfPhysics
B.Sc. Physics Mechanics
( ). Consider a wave of light starts from and at with speed . Let the wave
reaches a point P after time from and takes the time to reach at P from point . Then the
distance covered by light ray from point to point :
| |

( )
And the distance covered by light ray from point to point P:
| |

( )
Comparing these equations, we get:
( )
This is the fundamental equation of special theory of relativity given by Einstein in 1905.

Galilean Transformations Doesn’t Satisfy the Fundamental Equation of Relativity


Applying the values of from Galilean Transformation in Fundamental Equation of
Relativity
( )
( )
This is clearly impossible until . Hence Galilean Transformation fail to satisfy Fundamental
Equation of Relativity.

Lorentz Transformations Satisfy the Fundamental Equation of Relativity


Hence we need such transformations which satisfy Fundamental Equation of Special Theory
of Relativity. Such transformations are called Lorentz Transformation. These are given below:
( )

( )
{ }

51
Contact Us: aliphy2008@gmail.com, www.facebook.com/HomeOfPhysics
B.Sc. Physics Mechanics
Derivation of Lorentz Transformation
Consider two inertial frames of reference and . Frame is at rest and is moving with
velocity in the direction of increasing .
As relative velocity is along . Moreover, and coincides at
, so: and . So the fundamental equation of special theory of relativity
becomes:
( )
 As move along with relative velocity v with respect to ‘S’, so distance covered
by with respect to S after time will be .
 As O also appear to move along negative with relative velocity . So the distance
covered by with respect to after time is equal to .
These two requirements can be satisfied by putting:
( ) ( )
and
( ) ( )
In equation (1) and (2), if we know values of ‘ ’ and ‘ ’, then we can find the relations between
( ) and ( ), which satisfy equation (A).
To find ‘ ’ and ‘ ’, we put the value of from equation (1) in equation (2), we get:
[ ]
[ ( ) ]
( )

( )

[ ( )] ( )

Putting the values of and from equations (1) and (3) in equation (A):

[ ( )] [ ( )]

( ) [ ( ) ( ) ]

52
Contact Us: aliphy2008@gmail.com, www.facebook.com/HomeOfPhysics
B.Sc. Physics Mechanics

( ) ( )

( ) ( )

[ ( ) ] [ ( )] [ ]

This relation must hold for all values of and . So the coefficients of , and must be
zero separately. So we get three equations:

( ) ( )

( ) ( )

( )
From equation (6), we have:

( )

( )

( )

Now consider equation (5):

( )

( )

( )

53
Contact Us: aliphy2008@gmail.com, www.facebook.com/HomeOfPhysics
B.Sc. Physics Mechanics


Putting values of ‘ ’ and ‘ ’ in equation (1) and (2), we get:

( ) ( )

( ) ( )

Where is called Lorentz’s factor.


Now putting in equation (3), we get:


[ ( )]

[ ( { })]

[ ( )]

[ ( )]

( )

( ) ( )

54
Contact Us: aliphy2008@gmail.com, www.facebook.com/HomeOfPhysics
B.Sc. Physics Mechanics
Conclusion:
The set of transformation equations
( )

( )
{ }
is called Lorentz’s Transformation. When i.e., , the Lorentz Transformations transform

into Galilean transformation. Thus the Galilean Transformations is special case of Lorentz
Transformation.
Inverse Lorentz Transformation
The Lorentz Transformation equations are as follows:
( ) ( )
( )
( )

{ ( ) ( )}

From equation (4):

Putting the value of in equation (1):

[ ( )]

( )

( )

( )

( ) ( )
Now consider equation (1):
( )

55
Contact Us: aliphy2008@gmail.com, www.facebook.com/HomeOfPhysics
B.Sc. Physics Mechanics

Putting value of in equation (4), we get:

[ ( )]

( )

( )

( ) ( )

Hence the Inverse Lorentz Transformations are:


( )

( )
{ }
Important Note:
We can obtain the inverse Lorentz Transformations just by interchanging primed and unprimed
coordinates and replacing by – .
Transformation of Velocities
The equations of Lorentz Transformations can be used to get a relation between velocity of
a particle measured by an observer in frame and velocity of the same particle measured by an
observer in frame who is moving with velocity with respect to .
Suppose according to , particle moves from ( ) to ( ) and
according to , the particle moves from ( ) to ( ).
The -component of velocity measured by will be:

( )

By Lorentz Transformations, we have:


( )

56
Contact Us: aliphy2008@gmail.com, www.facebook.com/HomeOfPhysics
B.Sc. Physics Mechanics
( )
Also,

( )

( )

Putting values in equation (1):

( )
( )

( )

( )

The -component of velocity measured by will be:

( )

As
Putting values in equation (2), we get:

( )

( )

( )

( )

( )

57
Contact Us: aliphy2008@gmail.com, www.facebook.com/HomeOfPhysics
B.Sc. Physics Mechanics
Similarly,

( )

It should be noted that and even though and . This is another


difference between Lorentz and Galilean transformations.
Inverse Velocity Transformations
We can obtain inverse velocity transformation from equations of velocity transformation
simply by changing by – replacing primed coordinates with unprimed coordinates and vice versa.
So inverse velocity transformations are:

( )

( )

Lorentz Velocity Transformations under Non-Relativistic Limit

Under non-relativistic limit (i.e., for , we put ) the equations of velocity transformations

takes the form:

This set of equation is called Galilean Velocity Transformation. So under non-relativistic limits, the
Lorentz Velocity Transformation change into Galilean Velocity Transformation.
The Lorentz Velocity Transformation and Einstein’s 2nd Postulate
We can derive the result of Einstein’s 2nd postulate from Lorentz Velocity Transformations.
According to Einstein’s 2nd postulate, the speed of light is constant for all observers. So speed c
measured by an observer must also be measured to be c by any other observer.
Suppose the two observers observe a common event of passage of light beam along x-axis in
frame and . According to observer in , the velocity of light beam along x-axis and
. So, according to Lorentz Velocity Transformation, the velocity measured by :

58
Contact Us: aliphy2008@gmail.com, www.facebook.com/HomeOfPhysics
B.Sc. Physics Mechanics

( ) ( )

And

( ) ( )
( )

( ) ( )
( )

Therefore, velocities measured by are . So the observer also


measures the same speed. Hence the speed of light is same for all observers.

CONSEQUENCE OF SPECIAL THEORY OF RELATIVITY


Relativity of Time
Consider two frames of references and . is at rest and is moving with uniform velocity
with respect to . Suppose an event occurs at one and same place ‘x’ in frame .
 The duration of event measured by the observer in frame is .
 The duration of same event measured by the observer in frame is .
By using Lorentz Transformation:

( )

( )
Now

( ) ( )

( )

[( ) ( )] ( )

Because the event occurs at the same place, therefore: . Equation (1) takes the form:

[ ( )]

√ √

Since

59
Contact Us: aliphy2008@gmail.com, www.facebook.com/HomeOfPhysics
B.Sc. Physics Mechanics
So,
Hence the observer in frame will conclude that the clock in frame is slowed down i.e., time is
dilated.
Relativity of Length (Length Contraction)
Consider a rod lying at rest along x-axis in stationary frame S. Let the coordinates of its ends in this
frame be and . Then, length of the rod is called proper length and described as:

Let the length of the rod seen in moving frame moving with velocity be . Let the coordinates of
ends of the rod in frame of reference (FOR) are and . Then the length of the rod observed
with respect to will be:

It should be noted that the measurements are made simultaneously in both frames.
By Lorentz Transformation, we have
i. ( ) ( )
ii. ( ) ( )
Subtracting (1) and (2), we have
[ ]
[ ( )]
[ ]

Putting , because measurements are made simultaneously:

√ √

Therefore,

i.e., the length of the rod appears to be reduced in moving frame . This effect is called Length
Contraction.

Relativity of Mass
The Einstein’s Energy-Mass relationship is described as:
60
Contact Us: aliphy2008@gmail.com, www.facebook.com/HomeOfPhysics
B.Sc. Physics Mechanics

Here E, m & c represent energy, mass and speed of light respectively.


Differentiating both sides, we have:
( ) ( )
Let the work is done on the object by the force F which displaced it through distance dr. As the
work done on an object appears in the form of change in its energy , therefore, we can write:

By Newton’s second law of motion, the applied force on an object is equal to time rate of change of

linear momentum p i.e., . Therefore,

[ ( )]

[ ]

[ ]

( )
Comparing eq. (1) and (2), we get:

( )

( )

( )

( )

Integrating both sides, we have:

∫ ∫
( )

61
Contact Us: aliphy2008@gmail.com, www.facebook.com/HomeOfPhysics
B.Sc. Physics Mechanics
( )
∫ ∫
( )
( )

| | | ( )|

| | | ( ) |

( ) ( )

( ) ( )

( )

( )

This is the expression relativistic mass that describe mass variation with respect to velocity.
Equivalence of Mass and Energy (or Proof of )
Suppose a force acts on a body and as the result of this force, the body covers a distance in
direction of force. The work done by this force is:
⃗ ⃗⃗⃗⃗
( )
By Work-Energy Theorem, the work done on a body result in increase of its kinetic energy :
( )
Equating (1) and (2), we have:
( )
By Newton’s 2nd Law of Motion, the time rate of change of linear momentum of body is equal
applied force:

( )

62
Contact Us: aliphy2008@gmail.com, www.facebook.com/HomeOfPhysics
B.Sc. Physics Mechanics

Equation (3) becomes:

( )

( )
From relativistic mechanics,

( )

( )( ) ( )

( )

( )
( )

( )

( )

Putting values in equation (4), we get:


( )
( )
( )

Integrating bother sides:


( )
63
Contact Us: aliphy2008@gmail.com, www.facebook.com/HomeOfPhysics
B.Sc. Physics Mechanics
where A is constant of integration.
At , equation ( )
Now the equation (5) becomes:

( )
This equation shows that when , the body still possess some energy equal to , called rest
mass energy. Here is called total energy. Equation (6) takes the form:

This equation is called Einstein’s Energy-Mass Relationship.


Relativistic Energy
From Einstein’s Energy-Mass Relationship:

√ √

( )

( )
( )

The linear momentum a particle having mass moving with velocity is described as:

( )
( )

Subtracting equation (1) and (2):

( ) ( )

( )
( )

( )
( )

64
Contact Us: aliphy2008@gmail.com, www.facebook.com/HomeOfPhysics
B.Sc. Physics Mechanics


This is the expression of relativistic energy.
Sample Problem 6: What is momentum of a proton moving with speed of ?
Solution:
Rest mass of proton
Speed of Proton
Momentum
As

( )

√ ( )

65
Contact Us: aliphy2008@gmail.com, www.facebook.com/HomeOfPhysics
B.Sc. Physics Mechanics

WORK AND ENERGY


Work Done by the Constant Force
Consider a constant force F acts on a body and displaces it through a distance S in its own
direction. Then the work done is defined as the product of
magnitude of force and displacement:

Work W  F x  F x

However if the force makes an angle  with the


direction of motion of the body, then work is defined as
“the product of component of force along the line of motion and the magnitude of displacement”. In
this case, work is also defined as “the product of
magnitude of force and the component of
displacement along the direction of force. Thus
W   F cos   x

Or W  F  x cos  

W  F x cos   F .x
So, work is defined as the “Scalar product of force and displacement”.

If there are number of forces acting on the body, then work done is separately calculated for
each force. Then the net work will be the sum of work done by all separate forces.
Case 1. If   0
If the displacement is produced in the direction of the force i.e.,
  0 , then work done will be:
W  F .x
W  F x cos   F x cos 0
As cos 0  1
W  F x
Thus, when a horizontal force moves a body horizontally
or when a vertical force lifts a body vertically, then the work
66
Contact Us: aliphy2008@gmail.com, www.facebook.com/HomeOfPhysics
B.Sc. Physics Mechanics
done is the product of force and distance covered by the object.
Case 2. If   90
When the force has no componet in the direction of motion, then no work is done by the force.
W  F .x
W  F x cos   F x cos 90
As cos 90  0
W 0
Thus, if a person carrying a weight walks horizontally, then the
force exerted by the man is perpendicular to the horizontal
displacement. So, no work is done.
The examples of forces which don’t work are:
 Centripetal force
 Tension in the string of vibrating pendulum
 Weight and normal force don't work because they are
perpendicular to the displacement.
Case 3. If   180
When the force has a component opposite to the direction of displacement, then the work done by the
force
W  F .x
W  F x cos   F x cos180
As cos180  1
W   F x
The force of friction is opposite to the direction of motion, so the work done by the force of friction
on the object is zero.
Joule
The work done is a scalar quantity and its unit is Joule. Work is one joule when force of one
Newton acts on a body and displaces it through a distance of 1 meter in its own direction.
Note: The force is invariant and is independent of the choice of frame of reference. However,
displacement is not invariant and depends upon the frame of reference, in which the measurements of
displacement are carried out.
Work Done by a Variable Force (In One Dimension)

67
Contact Us: aliphy2008@gmail.com, www.facebook.com/HomeOfPhysics
B.Sc. Physics Mechanics
Let the force is in x-direction and its magnitude changes with respect to position. The
variation of force with displacement x is described in the figure.
Figure a shows the force is the function of displacement.
We have to calculate the work done from initial position
xi to final position x f .

For this we divide the displacement into N small intervals


each of length x .
Let F1 is the magnitude of force during the first interval,
then the work done by the force during the first interval is
approximately written as:
W1  F1x
This is the shaded area of first rectangle. Similarly for
other intervals:
W2  F2 x

W3  F3x
: : : : :
: : : : :
Wn  Fn x

The total work done from position xi to position x f is:

W  W1  W2  W3  ......  Wn

W  F1x  F2 x  F3x  ......  Fn x


N
W   Fi x
i 1

In order to obtain better results, we divide the displacement into a very large number of equal
intervals such that N   and x  0 . Hence
N N
W  lim  Fi x  lim  Fi x
N  x 0
i 1 i 1

xf

W  F x
xi

Which is area under the curve.

68
Contact Us: aliphy2008@gmail.com, www.facebook.com/HomeOfPhysics
B.Sc. Physics Mechanics
Explanation (Work Done by Variable Force)
A good example of one dimensional force is in the case of mass-spring system as shown in
the figure. External force Fapp is applied to the body towards

right, then
Fapp  kx

Where k is the spring constant and is the measure of


stiffness or softness of the spring. x is the displacement in
the of object from equilibrium position.
The restoring force FS exerted by the spring on the body is:

FS  kx
This is known as the Hook’s law.
Work done by the spring on the body,
when the body moves from its initial position
xi to x f will be:
xf

WS   F ( x).dx
xi
S

xf xf
1
 WS   (kx)dx   kx 2
xi
2 xi

1 2 1
 WS  kxi  kx f 2
2 2

 
When the spring is stretched xi  x f , work is

negative.

When the spring is compressed x i  xf ,


work is positive.
If the particle is moved from mean
position  x  0  i.e., xi  0 to x f  x , then the

work done by the spring on the body will be:


1 1
k  0   kx 2
2
WS 
2 2
1
WS   kx 2
2
Now the work done by the external force is:

69
Contact Us: aliphy2008@gmail.com, www.facebook.com/HomeOfPhysics
B.Sc. Physics Mechanics
x x
1
Wext   Fext ( x)dx   kx dx  kx 2
0 0
2

Work Done by a Variable Force (Two Dimensional Case)


In case of a variable force, its magnitude and direction may change so that the particle may
move along curved path.
To find out the work done, we divide the path into a large number of small displacements  s
. Each  s is along tangent to the path and points in the direction of motion. Work done during
displacement  s p is

Wp  Fp . s p  Fp cos p  s p
The total work done between the points i and f is calculated by adding the elements of work
for each segment Wi :
f f
W   Fp . s p   Fp cos  p s p
p i p i

If  s becomes infinitesimally small such that  s  0 , then  s can be replaced by dx and


summation by integration:
f f

W   F .ds   F cos  ds ---------- (1)


i i

As both F and  vary point to point, so F will have non-zero horizontal as well as vertical
components. Thus
F  Fxiˆ  Fy ˆj and ds  dxiˆ  dyjˆ
f

W   F .ds
i

 
W   Fxiˆ  Fy ˆj . dxiˆ  dyjˆ 
i

W   Fx dx  Fy dy ---------- (2)
i

Equation (1) and (2) are known as the line integral. In case of three dimensions:
f

W   Fx dx  Fy dy  Fz dz
i

70
Contact Us: aliphy2008@gmail.com, www.facebook.com/HomeOfPhysics
B.Sc. Physics Mechanics
Work-Energy Theorem
Statement:
The net work done by the forces acting on the particle is equal to the change in the kinetic
energy of the particle.
Derivation
When a net force acts on the body, it changes its state of motion by producing acceleration a
in it. Let the net force Fnet changes the velocity of the body from xi to x f , then

Wnet  Fnet ( x f  xi )

Wnet  ma.( x f  xi ) ---------- (1)

Fnet  ma ( Newton Second Law)


Using 3rd equation of motion:
v f 2  vi 2  2a( x f  xi )

a
v  v 
f
2
i
2

2 x  x  f i

Putting values in equation (1), we have:

Wnet  m.
 v  v  .( x
f
2
i
2

 xi )
2 x  x 
f
f i

v 2
 vi 2   1m
Wnet  m.
f

2 2
v f
2
 vi 2 
v 2
 vi 2   1m
Wnet  m.
f

2 2
v f
2
 vi 2 
1 1
Wnet  mv f 2  mvi 2
2 2
Wnet  K .E f  K .Ei  K .E

This is the mathematical form of work-energy theorem. This proof corresponds to the constant force
acting on the o object. However, this expression is not valid for the case of non constant force.
General Proof of Work Energy Theorem
If a non-constant force acts on the object in one dimension, then the word done by the force
on the object can be find out by using expression:
Wnet   Fnet dx ---------- (1)

dv dv dx dv dv
Now Fnet  m m  m v  mv , thus equation (1) becomes:
dt dx dt dx dx

71
Contact Us: aliphy2008@gmail.com, www.facebook.com/HomeOfPhysics
B.Sc. Physics Mechanics
dv
Wnet   mv dx
dx
vf

Wnet   mv dv
vi

vf
v2
Wnet m
2 vi

1
Wnet  m(v f 2  vi 2 )
2
1 1
Wnet  mv f 2  mvi 2
2 2
Wnet  K .E f  K .Ei  K .E

This is the expression of kinetic energy for the object under the action of non-constant force.
Power
The rate of doing work is called power. If an agent does work ‘ W ’ in time ‘ t ’, then the
average power is defined as the ration to total work done to the total time. It is described
mathematically as:
W
Pav  P 
t
If the power is variable, then the instantaneous power is given by the expression:
W dW
Pins  lim 
t 0 t dt
Watt
The SI unit of power is watt which can be defined as:
“If an agent does work of one joule of work per second,
the power of that agent will be 1 watt”
Question: Prove that P  F .v
dW
Proof: As P 
dt
Here the work done dW  F .d s , therefore

F .d s ds
P  F.
dt dt
ds
But v
dt
So P  F .v

72
Contact Us: aliphy2008@gmail.com, www.facebook.com/HomeOfPhysics
B.Sc. Physics Mechanics
CONSERVATION OF ENERGY

We come across two types of forces in our daily life:


 Conservative forces
 Non-conservative forces
Conservative Forces
If the work done by the force on the body depends upon the initial and final locations and is
independent of path taken by the body between the two points, then such a force is a conservative
force.
Or
If the net work done by the force on the body along the close path is zero, then such a force is
called conservative force.
Conservative forces are also distinguished by the ability to store energy only due
configuration of the system. This stored energy is called potential energy.
Examples of Conservative Forces
The Spring Force
Consider a block of mass m attached to a spring of spring constant k. The block is capable of
moving on horizontal frictionless table.
Figure 1. The external agent has displaced the
object from mean position (x = 0) to extreme
position (x = +A).
Figure 2. The external agent is suddenly removed at
t  0 and the spring begins to do work on the block.
As the object moves from x = +A to x = 0, the
1 2
spring does work W1  kA .
2
Figure 3. When the spring moves from x  0 to
x   A , the spring force reverses and spring acts to
slow down the block and does negative work.
1
W2   kA2
2
Figure 4. The object moves from spring moves
1 2
x   A to x  0 , and spring does work W3  kA .
2
Figure 5. The object moves from x  0 to x   A .

73
Contact Us: aliphy2008@gmail.com, www.facebook.com/HomeOfPhysics
B.Sc. Physics Mechanics
1
The spring does negative work W2   kA2 (as the block slows down).
2
The total work done by the spring force during the complete cycle (close path) is zero:
Total work WT  W1  W2  W3  W4

1 2  1 2 1 2  1 2
WT  kA    kA   kA    kA   0
2  2  2  2 
So, the spring force is conservative force.
Spring Force is Independent of Path Followed
A
Consider of a mass spring system. The block moves from x   A to x   along two different
2
paths.
A
Path 1. The block moves directly from x   A to x   . The work done by the spring:
2
A
A 
 2
1  A  
2 2
1 2
W1   (kx) dx   kx   k      A2 
A
2 A
2  2  

1  A2  1  3 A2 
 W1   k   A2    k  
2  4  2  4 

3kA2
 W1  
8
A
Path 2. From x   A to x   A and from x   A to x  
2
A

A 2
W2   (kx) dx   (kx) dx
A A

A
A 
1 1 2
W2   kx 2   kx 2
2 A 2 A

1  A  2
2
1 
 W2   k   A   A  k       A  
2
2

2   2  2 
 

1 1  A2 
 W2   k 0  k   A2 
2 2  4 

1  3 A2 
 W2   k  
2  4 

3kA2
 W2  
8
74
Contact Us: aliphy2008@gmail.com, www.facebook.com/HomeOfPhysics
B.Sc. Physics Mechanics
Conclusion: As W1  W2 , so work done is independent of path followed.

The Force of Gravity


Consider a ball of mass m is thrown upwards by an external agent. As the ball rises to height
y  h , the force of gravity mg acts downwards while the distance covered is upwards. So the work
done by the earth on the body is  mgh . The ball momentarily comes to rest and then falls from
y  h to y  0 , the work done by the gravity is  mgh .
The total work done along the round trip  mgh  mgh  0 . So the force of gravity or
gravitational force is a conservative force.
From the criterion that work done on a system by a conservative force along a closed path is
zero, we have:
B A

 F .d s   F .d s   F .d s  0
A B
path1 path 2

B A

 F .d s    F .d s
A B
path1 path 2

B B

 F .d s   F .d s
A A
path1 path 2

This shows that the work done on a system by a conservative force between two points is
independent of path.
*Electrostatic and Magnetic forces are conservative forces,
while the frictional force is an example of a non-conservative force.
Potential Energy
Potential energy is the energy of the configuration of the system and can only be defined for
conservative forces such as spring force or the force of gravity.
When configuration of a system undergoes a change, work is done by the conservative force.
Thus
U  W
The change in potential in a process is equal to the negative of work done by the conservative force.
If E is the mechanical energy of a conservative system, then by law of conservation of
energy:
U  K  E (constant)
Or  U  K   0

Where U and V are the potential energy and kinetic energy of the mechanical system.

75
Contact Us: aliphy2008@gmail.com, www.facebook.com/HomeOfPhysics
B.Sc. Physics Mechanics
One Dimensional Conservative System
When a one dimensional conservative force F(x) acts upon a body and as the result the object
moves from x0 to x , then the change in P.E. is given by the expression:
x
U  W    F ( x) dx ---------- (1)
x0

Equation (1) can also be described as:


x
U ( x)  U ( x0 )    F ( x) dx ---------- (2)
x0

If x0 is an arbitrary reference point, then potential energy function U can be obtained. If x0

is at infinity: U ( x0 )  U ()  0 , then the resulting function U ( x) can be used to calculate the P.E. at

the particular points in motion, say x1 and x2 .

In moving from x0 to x , the velocity of the particle changes from v0 to v . Hence according
to the Work Energy Theorem, the work done by the force is:
1 2 1
W  K  mv  mv0 2
2 2
From equation (1),
U  W
1 1 
 U ( x)  U ( x0 )    mv 2  mv0 2 
2 2 
1 1
 U ( x)  U ( x0 )   mv 2  mv0 2
2 2
1 1
 U ( x)  mv 2  U ( x0 )  mv0 2
2 2
1
 U ( x)  mv 2  E ---------- (3)
2
Where E is the mechanical energy of the system which depends upon depends upon the initial
position x0 and initial velocity v0 ; which have the definite values. So it is constant during motion
i.e., the mechanical energy E is constant.
Differentiating equation (2), we have:
x x
d d d
U ( x)  U ( x0 )    F ( x) dx    F ( x) dx
dx dx x0 x0
dx

dU ( x) dU ( x0 )
  F ( x)  0 as U ( x0 ) is a constant.
dx dx

76
Contact Us: aliphy2008@gmail.com, www.facebook.com/HomeOfPhysics
B.Sc. Physics Mechanics
dU ( x)
F ( x) 
dx
Thus
“Potential energy is a function of position whose negative derivative gives force”.
The Spring Force
Consider a mass-spring system. When the block is displaced a distance x from x0 , the
potential energy of the system is given by:
x
U ( x)  U ( x0 )    F ( x) dx
x0

Initially, the mass is at mean position i.e., x0  0 , so U ( x0 )  0 . And F ( x)   kx


x
U ( x)    ( kx) dx
x0

1 2
U ( x)  kx ---------- (1)
2
When the spring is compressed or stretched (i.e. either x is negative or positive), we have
same results.
Differentiating equation (1), we have:
dU ( x) d  1 2 
  kx   kx
dx dx  2 
dU ( x)
  kx
dx
dU ( x)
  F ( x)
dx
Suppose the block is stretched from x  0 to a distance xm , then the energy stored in the mass

1
spring system is kxm 2 . As the block is momentarily at rest, so its K.E = 0 . Hence the entire energy
2
is its P.E.
1
E kxm 2
2
From this, the instantaneous velocity is:

k
v ( xm 2  x 2 )
m

k
The speed of the bock is maximum at x  x0  0 : vmax  v0  xm
m
The speed of the bock is zero at x  xm : v  0

77
Contact Us: aliphy2008@gmail.com, www.facebook.com/HomeOfPhysics
B.Sc. Physics Mechanics
The Gravitational Force
For the ball-earth system, we choose y0  0 as a reference point at the surface of the earth, therefore

U ( y0 )  0
Using relation
y

U ( y )  U ( y0 )    F ( y ) dy
y0

Putting the values U ( y0 )  0 and F ( y )   mg


y

U ( y )    (mg ) dy
y0

U ( y )  mgy
Differentiating, we get:
dU ( y) d (mgy)
  mg
dy dy
dU ( y)
  mg  F ( y)
dy
dU ( y )
F ( y)  
dy
Using the law of conservation of mechanical energy (in y-direction), we have:
1 2
mv  U ( y )  E
2
1
Putting the values U ( y )  mgy and E  mv0 2 :
2
1 2 1
mv  mgy  mv0 2
2 2
v 2  2 gy  v0 2

v 2  v0 2  2 gy

This is the expression of velocity of block at height y in earth’s gravitational field.


One Dimensional Conservative System (Energy Method)
At each point, the force Fx on the any object is equal to the negative of the slope of the U(x) curve:

dU
Fx  
dx
Points x1 and x3 are stable equilibrium points. At each of these points, Fx is zero because the
slope of the U(x) curve is zero. When the particle is displaced to either side, the force pushes back
toward the equilibrium point. An analogous situation is a marble rolling around in a round-bottomed

78
Contact Us: aliphy2008@gmail.com, www.facebook.com/HomeOfPhysics
B.Sc. Physics Mechanics
bowl. We say that at points x1 and x3 is a point of stable equilibrium. More generally, any minimum
in a potential-energy curve is a stable equilibrium position.

The slope of the U(x) curve is also zero at points x2 and x4 , and these are also equilibrium
points. But when the particle is displaced a little to the right of either point, the slope of the U(x)
curve becomes negative, corresponding to a positive Fx that tends to push the particle still farther
from the point. When the particle is displaced
a little to the left, Fx is negative, again

pushing away from equilibrium. This is


analogous to a marble rolling on the top of
a bowling ball. Points x2 and x4 are called

unstable equilibrium points; any


maximum in a potential-energy curve is
an unstable.
the direction of a conservative force The direction of the force on a body is not determined by the
dU
sign of the potential energy U. Rather, it's the sign of Fx   that matters.
dx
If the total energy is E1 and the particle is initially near x1 , it can move only in the region

between xa and xb determined by the intersection of the E1 and U graphs (Fig. a). Again, U cannot

79
Contact Us: aliphy2008@gmail.com, www.facebook.com/HomeOfPhysics
B.Sc. Physics Mechanics
be greater than E) because K can't be negative. We speak of the particle as moving in a potential
well, and xa and xb are the turning points of the particle's motion (since at these points, the particle

stops and reverses direction). If we increase the total energy to the level E2 , the particle can move

over a wider range, from xc and xd . If the total energy is greater than E3 , the particle can "escape"
and move to indefinitely large values of x. At the other extreme, Eo represents the least possible total
energy the system can have.
Sample problem 4.
The potential energy function for the force between two atoms in a diatomic molecule can be
expressed approximately as:
a b
U ( x)  12
 6
x x
Where a and b are the positive constants and x is the distance between the atoms. Find (a) the
equilibrium separation between the atoms. (b) The force between atoms and (c) minimum energy
necessary to break the molecule apart (i.e., to separate the atoms fron equilibrium position to x   ).
Solution
The plot of potential energy U ( x) verses separation x is shown in the figure a and the plot of force
F verses x as shown in the figure b.

Let the equilibrium occurs between the atoms at x  xm at which U ( x) is minimum and is found
from:
 dU 
  0
 dx  x  xm
80
Contact Us: aliphy2008@gmail.com, www.facebook.com/HomeOfPhysics
B.Sc. Physics Mechanics
d  a b 
  12  6   0
dx  x x 

 12a  6  b   12a 6b 
  13  7    13  7   0
 x x   x x 

x7
Multiplying above equation by :
6
 2a b 
 6  0
 x 1
2a 2a
 6
 b  x6 
x b
1
 2a  6
xm   
 b 
The is the require expression for equilibrium separation between the atoms of diatomic molecule.
(b). The force corresponding to the minimum potential energy is:
dU d  a b  12a 6b
F     12  6   13  7
dx dx  x x  x x
This force is positive between x  0 and x  xm , the atoms are repelled from one another and the

force is directed towards increasing x.. When the force is negative from x  xm and x   , the atoms
are attractive to one another (this force is directed towards decreasing x).
At x  xm , the force is zero. This is the equilibrium point and is stable equilibrium.

(c). The minimum energy required to break the molecule into separate atoms is called dissociation
energy Ed we can separate the atoms to x   when U  0 , whenever E  0 . The minimum energy
required corresponds to E  0 i.e., the atoms are infinitely separated ( U  0 ). The energy added to
the molecule in its equilibrium state to rise its energy from negative value to zero is called its
dissociation energy Ed.
U ( xm )  Ed  0
a b
Ed  U ( xm )   12

x
m xm 6
1
 2a  6
Substituting xm    , we get:
 b 
b2
Ed 
4a

81
Contact Us: aliphy2008@gmail.com, www.facebook.com/HomeOfPhysics
B.Sc. Physics Mechanics
Two and Three Dimensional Conservative Systems
In three dimensions, the potential energy can be written as U ( x, y, z ) . So the equation
x
U  U ( x)  U ( x0 )    F ( x) dx in three dimensions is
x0

x y z
U    Fx dx   Fy dy   Fz dz
x0 y0 z0

Where U is the change in potential energy for the system when the particle moves from
point ( x0 , y0 , z0 ) to ( x, y , z ) . Fx , Fy and Fz are the components of conservative force

F ( r )  F ( x, y , z ) .
1 2 1
The relation mv  U ( x)  mv0 2  U ( x0 ) in three dimensional motion is:
2 2
1 2 1
mv  U ( x, y, z )  mv0 2  U ( x0 , y0 , z0 )
2 2
In vector notation:
1 1
mv .v  U (r )  mv0 .v0  U (r0 )
2 2
Where v .v  vx 2  v y 2  vz 2 and v0 .v0  v0 x 2  v0 y 2  v0 z 2

In terms of mechanical energy:


1 2
mv  U ( x, y, z )  E
2
Also
dU U
Fx   
dx x
dU U
Fy   
dy y
dU U
Fz   
dz z
Putting values in equation
F (r )  Fxiˆ  Fy ˆj  Fz kˆ

U ˆ U ˆ U ˆ
F (r )   i j k
x y z
   ˆ  ˆ
F (r )    iˆ  j
 z 
k U
x  y
F (r )  U

82
Contact Us: aliphy2008@gmail.com, www.facebook.com/HomeOfPhysics
B.Sc. Physics Mechanics
So, the conservative force is equal to the negative of gradient of potential energy U ( x, y, z ) .
Conservation of Energy in System of Particles
Energy can be transformed from one kind to another in an isolated system, but it cannot be created
or destroyed: the total energy of the system remains constant.
Explanation
Consider a block-spring system. The block is placed on a table and a frictional force is present
between the block and the table. There are two transfer of energy through the system boundary: the
positive conservative work WS done on the block by the spring and the negative work W f done on

the block by the frictional force exerted by the table. For this system, the conservation of energy is
written as

83
Contact Us: aliphy2008@gmail.com, www.facebook.com/HomeOfPhysics
B.Sc. Physics Mechanics

COLLISIONS

Collision
Collision is an interaction between the bodies that occurs in time t that is negligible to the
time during which we are observing the system. We can also characterize a collision as an event in
which the external forces that may act on the system are negligible compared to impulsive collision
forces.
Note:
Collision does not mean, necessarily, physical contact between two bodies.
Elastic Collision
A collision in which the momentum as well as kinetic energies of the colliding bodies is the
same before and after the collision.
Kinetic energy is not transformed to the other forms of energy such as internal energy, light,
heat ad sound energy etc. also fragmentation of the colliding bodies don't take place.
Inelastic Collision
A collision in which linear momentum of colliding bodies is conserved, but kinetic energy is
not conserved.
Some of the kinetic energy may be converted into heat, light and sound etc. also
fragmentation of colliding bodies may take place.
Impulsive Forces
A force that acts for a very short interval of time during the collision is called impulsive
force. For example when a ball is hit with a bat, impulsive
force is applied.
Impulse and Momentum
Consider a collision, in which the impulsive force
F(t) is applied on the body. The magnitude of the forces is
shown in the figure.
The collision begins at time ti and ends at time t f . The force

is zero before and after collision.


According to the Newton’s 2nd law
dP
F
dt
So, the change in momentum d P of the particle in time dt during which the force acts on it is

84
Contact Us: aliphy2008@gmail.com, www.facebook.com/HomeOfPhysics
B.Sc. Physics Mechanics
d P  F dt
Integrating this equation between the initial and final conditions
Momentum Pi at time ti

Momentum Pf at time t f
Pf tf

 d P   F dt
Pi ti

tf

P f  Pi   F dt ----------- (1)
ti

Impulse of a force J is defined as product of force and time:


tf

J   F dt ---------- (2)
ti

Combining equation (1) and (2), we get:


J  P f  Pi
This is Impulse-Momentum Theorem which states that:
The impulse of the net force acting on a particle during a given time interval is equal to the
change in momentum of the particle during that interval.

Both the impulse and momentum are vector quantities having same units and dimensions.
Impulse-Momentum theorem is very similar to the work energy
theorem. The work-energy theorem is a scalar equation dealing change
in the kinetic energy of the particle while Impulse-Momentum theorem
is a vector equation dealing with the change of momentum of the
particles.
The magnitude of impulse of force is represented by the area
under the F(t) curve. The same area is represented by the rectangle of
width t and height F . Here F is the magnitude of the average force,
then
J  F t
Conservation of Momentum during Collision

Consider the collision between two particles


of masses m1 and m2 as shown in the figure.

85
Contact Us: aliphy2008@gmail.com, www.facebook.com/HomeOfPhysics
B.Sc. Physics Mechanics
During brief collision these particles exerts large force on one another.
Let
F 12 is the force exerted on the particle 1 on particle 2.
F 21 is the force exerted on the particle 2 on particle 1.
Let the collision occurs for a time interval t  t f  ti . The change in momentum of mass m1 is
tf

 P1   F 12 dt  F 12 t ---------- (1)
ti

Similarly, the change in momentum of mass m2 is


tf

 P 2   F 21 dt  F 21 t ---------- (2)
ti

As F 12 and F 21 are equal but opposite, then


F 12   F 21
Then from equation (1),
 P1  F 12 t   F 21 t   P 2

  P1   P 2  0
So, the total change on momentum  P1   P 2   P is zero. Hence we conclude that
If there is mo external forces, the total momentum of the two particle
system is not changed by collision.
The is the law of conservation of linear momentum, so
Momentum of the system before collision = Momentum of the system after collision
Elastic Collision in One Dimension
Consider two spherical objects of an isolated system having masses m1 and m2, moving with
initial velocities v1i and v2i , respectively, along the
same direction (say x-axis). After collision, their
velocities are v1 f and v2 f as shown in the figure:

As the collision is elastic, then by law of


conservation of linear momentum, we have:
m1v1i  m2 v2i  m1v1 f  m2v2 f ---------- (1)

m1v1i  m1v1 f  m2 v2 f  m2 v2i

m1  v1i  v1 f   m2  v2 f  v2i  ---------- (2)

From the conservation of kinetic energy:

86
Contact Us: aliphy2008@gmail.com, www.facebook.com/HomeOfPhysics
B.Sc. Physics Mechanics
1 1 1 1
m1v1i 2  m2v2i 2  m1v1 f 2  m2v2 f 2
2 2 2 2
m1v1i 2  m2v2i 2  m1v1 f 2  m2v2 f 2

m1v1i 2  m1v1 f 2  m2v2 f 2  m2v2i 2

 
m1 v1i 2  v1 f 2  m2 v2 f 2  v2i 2  
m v
1 1i  v1 f  v
1i  v1 f   m2  v2 f  v2i  v2 f  v2i  ---------- (3)

Dividing equations (2) and (3)


m1  v1i  v1 f  v
1i  v1 f   m v2 2f  v2i  v2 f  v2i 
m1  v1i  v1 f  m2  v2 f  v2i 

v 1i  v1 f    v2 f  v2i  ---------- (4)

v1i  v2i  v2 f  v1 f

 v1i  v2i     v1 f  v2 f 
vrel ,i  vrel , f

So, the relative velocity of approach before collision is equal and opposite to the relative
velocity of separation after collision. From equation (4), we have:
v2 f  v1i  v1 f  v2i ---------- (5)

Putting the value of v2 f in equation (1)

m1v1i  m2v2i  m1v1 f  m2  v1i  v1 f  v2i 

m1v1i  m2 v2i  m1v1 f  m2v1i  m2v1 f  m2v2i

m1v1i  m2 v2i  m2 v2i  m2v1i  m1v1 f  m2v1 f

 m1  m2  v1i  2m2v2i   m1  m2  v1 f
 m1  m2  v1 f   m1  m2  v1i  2m2v2i
 m  m2   2m2 
v1 f   1  v1i    v2i ---------- (a)
 1
m  m2   1
m  m2 

Putting this value of v1 f in equation (5)

 m  m2   2m2 
v2 f  v1i   1  v1i    v2i  v2i
 m1  m2   m1  m2 
 m  m2   2m2 
v2 f  1  1  v1i    1 v2i
 m1  m2   m1  m2 

87
Contact Us: aliphy2008@gmail.com, www.facebook.com/HomeOfPhysics
B.Sc. Physics Mechanics
 m  m2  m1  m2   2m2  (m1  m2 ) 
v2 f   1  v1i    v2i
 m1  m2   m1  m2 
 2m1   m2  m1 
v2 f    v1i    v2i ---------- (b)
 m1  m2   m1  m2 
Special Cases:
Case 1. Equal Masses
When the colliding balls have equal mass, then by putting m1  m2 in equations (a) and (b),
we have:
v1 f  v2i

v2 f  v1i

Thus, if the balls of same masses collies each other, they will interchange their velocities after
collision.
Case 2. Equal Masses and Target Particle at Rest
Putting m1  m2 , v2i  0 in equation (a) and (b), we have:

v1 f  0

v2 f  v1i

Thus, the ball of mass m1 , after collision, will come to stop and m2 will takes of the velocity of m1 .

Case 3: When a Light Body Collides with the Massive Body at Rest.
When the mass of the target is much greater than that of incident particle i.e., m2 m1 , then

we have m1 0 ( m1 is negligible). For the present case, putting m1  0 and v2i  0 in equations (a)
and (b):
v1 f  v1i

v2 f  0 Thus, the body of mass m1 will bounce back with the same velocity while m2 will

remain stationary.
Case 4: When a Massive Body Collides with the Light Stationary Body.
When the incident particle is much massive as compared to the target m1 m2 , then we have

m2 0 ( m2 is negligible). For the present case, putting m2  0 and v2i  0 in equations (a) and (b):

v1 f  v1i

v2 f  2v1i

Hence the massive body will move with same velocity, while the lighter target at rest moves
at twice the speed of massive projectile.

88
Contact Us: aliphy2008@gmail.com, www.facebook.com/HomeOfPhysics
B.Sc. Physics Mechanics
Inelastic Collision
Consider an inelastic collision in which the kinetic energy is not conserved, however
momentum is conserved.
Let in a completely inelastic collision, both particles stick together after collision and move
with common velocity v f . So there is only on unknown v f and the momentum equation is

sufficient.
m1v1i  m2vi 2   m1  m2  v f

 m1   m2 
 vf    v1i    v2i ---------- (1)
 m1  m2   m1  m2 
When m2 is initially at rest i.e., v2i  0 , then

 m1 
vf    v1i
 m1  m2 
The equation (1) can also be applied equally well in reverse process. So that a particle of mass M
moving with velocity V splits up into two particles m1 and m2 ( M  m1 ) moving in opposite
direction. So,
MV  m1v1i  m2vi 2 ---------- (2)
Special case:
If the original particle is at rest i.e., V  0 , then equation (2) will become:
0  m1v1i  m2vi 2
v1i m
 2
vi 2 m1
So, the more massive particle moves with smaller velocity and vice versa. The two particles move in
opposite directions.
Sample Problem 2.
By what fraction is the kinetic energy of neutron (mass m1 ) decreased in a head on elastic

collision with an atomic nucleus (mass m2 ) initially at rest? Find the fractional decrease in kinetic
energy of a neutron when it collides in this way with a lead nucleus, a carbon nucleus and a hydrogen
m 
nucleus. The ratio of nuclear mass to neutron mass  2  is 206 for lead, 12 for carbon and 1 for
 m1 
hydrogen.
Solution.
1
Initial KE of neutron Ki  m1v12i
2

89
Contact Us: aliphy2008@gmail.com, www.facebook.com/HomeOfPhysics
B.Sc. Physics Mechanics
1
Final KE of neutron K f  m1v12f
2
Fractional decrease in KE:
1 1
Ki  K f m1v12i  m1v12f
2 2
Ki 1
m1v12i
2

Ki  K f v12i  v12f v12f


   1 ---------- (1)
Ki v12i v12i
The velocity of neutron after collision with massive target at rest:
 m  m2  v f  m1  m2 
vf   1  v1i    
 m1  m2  v1i  m1  m2 
2
 m  m2 
v 2f
 2  1 
vi  m1  m2 

Putting value in equation (1), we have


2
Ki  K f  m  m2 
 1  1 
Ki  m1  m2 
Ki  K f m12  m22  2m1m2
  1
Ki m12  m22  2m1m2


Ki  K f

m12  m22  2m1m2  m12  m22  2m1m2  
Ki m  m  2m1m2
2
1
2
2

Ki  K f m12  m22  2m1m2  m12  m22  2m1m2


 
 m1  m2 
2
Ki

Ki  K f 4m1m2
 
 m1  m2 
2
Ki

m2
(b) For lead  206  m2  206m1
m1

Ki  K f 4m1  206m1  4(206)m12


 
 m1  206m1   207m1 
2 2
Ki

Ki  K f
 0.02  2%
Ki
m2
For carbon  12  m2  12m1
m1

90
Contact Us: aliphy2008@gmail.com, www.facebook.com/HomeOfPhysics
B.Sc. Physics Mechanics
Ki  K f 4m1 12m1  4(12)m 2
  1

 m1  12m1  13m1 
2 2
Ki

Ki  K f
 0.28  28%
Ki
m2
For hydrogen  1  m2  1m1
m1

Ki  K f 4m1 1m1  4(1)m12


 
 m1  1m1   2m1 
2 2
Ki

Ki  K f
 1  100%
Ki
So, the material containing high content of hydrogen such as paraffix or water would be good
moderation in a nuclear reactor.
Sample Problem 3.
A ballistic pendulum is a device that as used to measure the speeds of bullets before
electronic timing devices were available. It consists of a large block of wood of mass M hanging
from two long pairs of cords. A bullet of mass m is
fired into the block and comes quickly to rest relative
to the block. The block + bullet combination swings
upwards, its center of mass rising a vertical distance h
before the pendulum comes momentarily to rest at the
end of its arc. Take the mass of the block to be M = 5.4
kg and the mass of the bullet to be 9.5 g.

(a) What is the initial speed of the bullet if


the block rises to a height of h = 6.3 cm? (b) What
is the initial K.E of the bullet? How much of this
energy remains as mechanical energy of swing
pendulum.
Solution.
The bullet and the block stick together in
an inelastic collision. By law of conservation of
momentum:
mv  M (0)   m  M V ---------(1)

Where

91
Contact Us: aliphy2008@gmail.com, www.facebook.com/HomeOfPhysics
B.Sc. Physics Mechanics
v  Velocity of the bullet before impact
V  Velocity of combination (bullet + block) after impact

Mechanical (kinetic) energy is not conserved during the collision of bullet with block, it is conserved
in the swinging pendulum after the impact.

K.E. of the combination at bottom = P. E. of combination at height h


1
 M  m V 2   M  m  gh
2
1 2
V  gh ---------- (2)
2
mv
From equation (1) V  , put in equation (2):
m  M 
2
1  mv 
   gh
2 mM 
2
1 m  2
  v  gh
2 mM 

 mM 
2

v  2
2
 gh
 m 

 mM 
2

v  2 gh
 m 

 5.4  0.0095 
2

v  2(9.8)(0.63)
 0.0095 
v  630 ms 1
(b) Initial K.E. of bullet
1 2 1
Kb  mv  (0.0095)(630) 2
2 2
Kb  1900 J

The mechanical energy of the  P.E. of swinging pendulum 


 = 
 swinging pendulum   at the top of its swing 
E  ( M  m) gh

E  (5.4  0.0095)  9.8 (0.063)

E  3.3 J

92
Contact Us: aliphy2008@gmail.com, www.facebook.com/HomeOfPhysics
B.Sc. Physics Mechanics
3.3
So only or 0.2 % of the original K.E of the bullet is transferred to mechanical energy
1900
of the pendulum. The rest energy is stored insider the pendulum block as the internal energy or
transferred to environment, for example as heat and sound.
Two Dimensional Collision (Elastic)
When the two objects don't collide head on manner, then they move in different direction
which doesn’t coincide with the original direction of motion. This is called oblique collision.
Consider an incident particle (projectile) of mass m1 moves with velocity v1i in positive x-direction

towards a particle (target particle) of mass m2 initially at rest i.e., v2i  0 .


The perpendicular distance ‘b’ between the line of motion incident particle and a line through
m2 is called the impact parameter. If b  0 , it is head on collision. If b  0 , the collision is oblique
or glancing.

Figure shows the path of two objects. The direction of velocity vectors v1 f and v2 f are

shown by 1 and 2 respectively.


Applying the law of conservation of linear momentum.
For x-component;
pix  p fy

m1v1i  0  m1v1 f cos 1  m2 v2 f cos 2 ---------- (1)

For y-component;
piy  p fy

0  0  m1v1 f sin 1  m2 v2 f sin 2 ---------- (2)

If the collision is elastic, then by law of conservation of KE


1 1 1
m1v12i  0  m1v12f  m2v22 f ---------- (3)
2 2 2

93
Contact Us: aliphy2008@gmail.com, www.facebook.com/HomeOfPhysics
B.Sc. Physics Mechanics
Here known quantities are m1 , m2 and v1i . And the unknown quantities are v1 f , v2 f , 1 and 2 .

But only three equations are there, which cannot determine there four unknowns. So, by applying an
additional restrictions; say a particular angle  , we can find the remaining three unknowns from
three equations (1), (2) and (3).
Sample Problem 4.
A gas molecule having a speed of 322 ms-1 collides elastically with another molecule of the
same mass which is initially at rest. After the collision, the first molecule moves at an angle of 30º to
its initial direction. Find the speed of each molecule after collision and the angle made with the
incident direction by the recoiling target molecule.
Solution.
m1  m2

v1i  322 ms 1

v2i  0

1  30
By the law of conservation of momentum: Along x-axis
m1v1i  0  m1v1 f cos 1  m2 v2 f cos 2

Putting m1  m2 we have:

v1i  v1 f cos 1  v2 f cos 2 ---------- (1)

By the law of conservation of momentum: Along y-axis


0  0  m1v1 f sin 1  m2 v2 f sin 2

Putting m1  m2 we have:

v1 f sin 1  v2 f sin 2 ---------- (2)

By law of conservation of KE
1 1 1
m1v12i  0  m1v12f  m2v22 f
2 2 2
Putting m1  m2 we have:

v12i  v12f  v22 f ---------- (3)

Now we eliminate 2 . By equation (1):

v1i  v1 f cos 1  v2 f cos 2 ---------- (4)

Squaring and adding equation (2) and (4):

v  v1 f cos 1   v12f sin 2 1  v22 f cos 2 2  v22 f sin 2 2


2
1i

94
Contact Us: aliphy2008@gmail.com, www.facebook.com/HomeOfPhysics
B.Sc. Physics Mechanics
v1i  v cos 1  2v1i v1 f cos 1  v1 f sin 1  v
2 2
1f
2 2 2 2
2f  cos 
2
2  sin 2
2

 
v12i  v12f cos 2 1  sin 2   2v1i v1 f cos 1  v22 f

v12i  v12f  2v1i v1 f cos 1  v22 f ---------- (5)

From equation (3):


v12i  v12f  v22 f ---------- (6)

Comparing equations (5) and (6):


v12i  v12f  2v1i v1 f cos 1  v12i  v12f

v12f  2v1i v1 f cos 1  v12f

2v12f  2v1i v1 f cos 1

v1 f  v1i cos 1

v1 f  (322) cos 30

v1 f  270 ms 1

Putting this value in equation (3), we have


v12i  v12f  v22 f

 322   279   161 ms 1


2 2
v2 f 

From equation (2):


v1 f sin 1  v2 f sin 2

v1 f
sin 2  sin 1
v2 f

279
sin 2  sin 30  0.866
161
2  sin 1  0.866 

2  60
Inelastic Collision in Two Dimensions
Consider a completely inelastic collision in which two bodies of masses m1 and m2 moves with

velocities v1i and v2i collide with each other. These bodies meet and stick together at origin

95
Contact Us: aliphy2008@gmail.com, www.facebook.com/HomeOfPhysics
B.Sc. Physics Mechanics
ROTATIONAL DYNAMICS

An overview of Rotational Dynamics


For linear motion, dealing with problems of dynamics, we have
Force = Mass * Acceleration
F  ma
Now we analyze the rotational motion of rigid bodies about a certain axis of rotation.
Consider a certain force is applied at a certain location to a rigid body free to rotate about a
particular axis, the resulting motion depends upon the location of the application of force. A given
force applied a one location may produce difference rotation at some other location. This quantity
which takes into account both the magnitude of the force and the location and direction in which it is
applied is called torque.
We regard force as push or a pull, similarly we can regard torque as twist.
The effort required to put a body into rotation depends on the distribution of mass of the body
about the axis of rotation. If the mass is closer to the azis of rotation, it is easier to rotate a body and
vice versa. The inertial quantity that takes into account the distribution of body’s mass is called
rotational inertia or moment of inertia. Unlike mass, rotational inertia is not an intrinsic property of
the body.
The equation, identical with equation (1), for rotational dynamics is:
Torque = Moment of Inertia * Angular Acceleration
  I
Relationship between Linear and Angular Variables:
Linear Motion Angular Motion
Linear Displacement x Angular Displacement 
dx d
Linear Velocity v  Angular Velocity  
dt dt
dv d
Linear Acceleration a  Angular Acceleration  
dt dt
dp dL
Force F  ma or F  Torque   I or  
dt dt
Work W   Fdx Work W    d
1 2 1 2
Kinetic Energy K  mv Kinetic Energy K  I
2 2
Power P  F v Power P   
Linear Momentum p  mv Angular Momentum L  I 
Linear Impulse = Ft Angular Impulse =  t
Mass (Translational Inertial) m Rotational Inertia I
96
Contact Us: aliphy2008@gmail.com, www.facebook.com/HomeOfPhysics
B.Sc. Physics Mechanics
Kinetic Energy of a Rigid Body

Consider a rigid body is spinning along the axis of rotation with


uniform angular velocity . Let the object consist of n particles having
masses , which are at distances from
axis of rotation.

The rotational kinetic energy for particle of mass

The rotational kinetic energy for particle of mass

The rotational kinetic energy for particle of mass

Now the total rotational kinetic energy acting on the rigid body is described as:

Since the body is rigid, so all the masses will rotate with same angular velocity ,

( )
(∑ )
Where ∑ is the moment of inertia of
the rigid body.
Parallel Axis Theorem
It states that
The rotational inertial of any body about
any axis is equal to the sum of rotational inertia
about a parallel axis through the centre of mass
and the product of mass of body and square of
distance between two axis”
Mathematically, it is described as:
I  I CM  Md 2

Where
I  Rotational inertia about an arbitrary axis

97
Contact Us: aliphy2008@gmail.com, www.facebook.com/HomeOfPhysics
B.Sc. Physics Mechanics
ICM  Rotational inertia about the parallel axis through the center of mass
M  Mass of the body
d  Perpendicular distance between the axes
Proof
Consider a thin plane slab in xy-plane. The plane can be regarded as the collection of particles each
having mass mi . Now we calculate the rotational inertia of the slab about z-axis passing through
origin O and perpendicular to the slab.
 xi , yi   Co-ordinates of mi w.r.t origin

 xi ', yi '  Co-ordinates of mi w.r.t center of mass

 xcm , ycm   Co-ordinates of center of mass w r t origin

From figure,
xi  xi ' xcm

yi  yi ' ycm
The rotational inertia about the axis through O is


I   mi xi 2 , yi 2 
I   mi  xi ' xcm    yi ' ycm  
2 2
 

I   mi  xi '2  xcm 2  2 xi ' xcm  yi '2  ycm 2  2 yi ' ycm 

   
I   mi xi '2  yi '2   mi xcm 2  ycm 2  2 xcm  mi xi ' 2 ycm  mi yi '

But m x '  0  m y '


i i i i

xcm 2  ycm 2  d 2

m x '  y '   I
i i
2
i
2
CM

 I  I CM  Md 2

Which is the mathematical form of parallel axis theorem.


Rotational Inertia for Solid Bodies
If we take very small mass tending to zero, then,

98
Contact Us: aliphy2008@gmail.com, www.facebook.com/HomeOfPhysics
B.Sc. Physics Mechanics
Rotational Inertial of Thin Uniform Rod
Figure shows a thin uniform rod of mass M, length L and area of cross-section A. we want to
find out the moment of inertia I about an axis passing through its center O.
∫ -------------- (1)
If is volume mass density, then

Consider the rod is placed along x-axis, therefore:

The equation (1), will become:

| |

[( ) ( ) ]

[ ]

[ ]

99
Contact Us: aliphy2008@gmail.com, www.facebook.com/HomeOfPhysics
B.Sc. Physics Mechanics
Rotational Inertia of Thin Uniform Rod about a Perpendicular Axis Passing Through Its Edge
Figure shows a thin uniform rod of mass M, length L and area of cross-section A.
We want to find out the moment of inertia I about a perpendicular axis passing through its edge. As
∫ -------------- (1)
If is volume mass density, then

Consider the rod is placed along x-axis, therefore:

The equation (1), will become:


L
M
I   x2 . dx
0
L
L
M 2
L 0
I x dx

L
M x3
I
L 3 0

M  L3 
I   0
L3 
M  L3 
I  
L3
1
I  ML2
3
Rotational Inertia of (i) A Hollow Cylinder (ii) A Solid Cylinder about Axis of Symmetry
(Cylindrical Axis)
Figure shows annular cylinder of mass M, length L with inner and outer radii R1 and R2 ,

respectively. Take a cylindrical shell of radius r and thickness dr .


Surface area of cylindrical shell ds  2 rL
Volume of cylindrical shell dv  2 rLdr
Mass of cylindrical shell dm   2 rLdr  

Rotational inertia of the annular cylinder is

100
Contact Us: aliphy2008@gmail.com, www.facebook.com/HomeOfPhysics
B.Sc. Physics Mechanics

I   r 2 dm
R2

I  r .  2 rLdr  
2

R1

R2

I  2 L r dr
3

R1

R2

I  2 L  r 3dr
R1

R2
r4
I  2 L
4 R1

1
I   L R2 4  R14
2
 
1
I   L R2 2  R12
2
  R 2
2
 R12 
I
1
2
 
 R2 2  R12  L  R2 2  R12  
 
Here M   R2 2  R12  L = Mass of cylindrical shell

I
1
2

M R2 2  R12  ------------ (1)

Rotational Inertia of A Solid Cylinder


For a solid cylinder, R1  0 and R2  R . By putting the values in

equation (1), we have:

I
1
2

M R 2  02 
1
I MR 2
2
Rotational Inertia of A Hollow Cylinder
For a hollow cylinder, R1  R2  R . By putting the

values in equation (1), we have:

I
1
2

M R2  R2 
I
1
2
M 2R2  
I  MR2

101
Contact Us: aliphy2008@gmail.com, www.facebook.com/HomeOfPhysics
B.Sc. Physics Mechanics
Rotational Inertia of (i) A Disk (ii) A Hoop (Ring) about Cylindrical Axis
Consider a disk of inner and outer radii R1 and R2 , respectively. Let  is surface mass
density.
Consider a circular strip of radius r and breadth dr with in the material.
Surface area of circular strip ds  2 rdr
Mass of circular strip ds   ds  2 rdr
Rotational inertia of strip is:
I   r 2 dm
R2

I  2 r dr
3

R1

R2

I  2  r 3dr
R1

R2

I  2  r 3dr
R1

R2
r4
I  2
4 R1

1

I   R2 4  R14
2

1

I   R2 2  R12
2
 R2
2
 R12 
I
1
2    
 R2 2  R12   R2 2  R12 
 
Here M   R2 2  R12  = Mass of cylindrical shell

I
1
2

M R2 2  R12  ------------ (1)

Rotational Inertia of A Solid Disk


For a solid disk, R1  0 and R2  R . By putting the values in equation (1), we have:

I
1
2

M R 2  02 
1
I MR 2
2
Rotational Inertia of A Hollow Cylinder
For a hoop or a ring, R1  R2  R . By putting the values in equation (1), we have:

102
Contact Us: aliphy2008@gmail.com, www.facebook.com/HomeOfPhysics
B.Sc. Physics Mechanics

I
1
2

M R2  R2 
I  MR2
Rotational inertia of a spherical shell
Fig. shows a thin spherical shell of radius R, mass m and surface mass density  . In fig
Rsin  is the radius of the circular ring, d  angular width and Rd  linear width of the ring.

Circumference of the ring = 2 R


Surface area of the circular strip ds =(2 R sin  )( Rd  ) = 2 R 2 sin  d 

Mass of circular strip dm =  ds= 2 R 2  sin  d 


Rotational inertia of the circular strip about the diameter of the shell

dI = r 2 dm   R sin   2 R 2  sin  d 
2

dI = 2  R 4 sin 3  d
Rotational inertia of the shell is

I   dI   r 2 dm   2  R 4 sin 3  d
0


= 2  R  sin  d  ---------- (1)
4 3

Here
  
3 2 2

 sin  d   sin  sin    1  cos  sin  d 
0 0 0


=  sin   cos 2    sin   d 
0
 
=  sin  d   cos 2    sin   d
0 0

103
Contact Us: aliphy2008@gmail.com, www.facebook.com/HomeOfPhysics
B.Sc. Physics Mechanics

 cos 
=  cos  0 
3 0

 1 1
=   cos     cos 0         2  
2 4
 3 3 3 3
Hence equation (1) will becomes:
4 2
I = 2  R 4 .  ( 4 R 2  ) R2
3 3
2
I MR 2 where M  ( 4 R 2  )
3
Rotational inertia of a solid sphere about its diameter
Fig, shows a solid sphere of radius R, mass M volume mass density  .Take a spherical shell
of radius r and width dr with in sphere.
2 2 2
dI  r * mass  r 2 * dm
3 3
mass of shell volume of shell

mass of sphere volume of sphere

dm 4 r 2 dr

M 4
 R3
3
3r 2
dm= Mdr
R3
So
2 2 3r 2
dI  r * 3 Mdr
3 R
2M 4
dI  r dr
R3
The moment of inertia of the sphere is the sum of moment of inertia of the various shells.
R
2M
I   dI  3  r 4 dr
R 0

2M  R5  2
 3 
 0  MR 2 `
R 5  5
Hence
2
I= MR 2
5

104
Contact Us: aliphy2008@gmail.com, www.facebook.com/HomeOfPhysics
B.Sc. Physics Mechanics
UNIVERSITY OF SARGODHA
B.A. / B.Sc. 1st Annual Exam 2017
Physics Paper: A
Time Allowed: 3 Hours Maximum
Marks: 50
Note: Attempt any five questions in all, selecting at least two questions from each part. All questions carry
equal marks.
SECTION-I
Q # 1. (a) What is special theory of relativity? State its basic postulates. Also apply it to calculate relativistic
momentum of a system of two particles.
(b) The most energetic proton detected in the cosmic rays coming to earth from space had kinetic energy of
. What were the proton's Lorentz factor γ and speed v (both relative to the ground based detector)?
Q # 2. (a) Describe three Kepler's laws about the motions of planets and satellites in detail.
(b) A satellite, orbiting at an altitude of two earth's radii above its surface, launches an equipment canister of
mass m towards the earth's center with a speed of 525 m/s. With what speed do the canister enter the earth's
atmosphere (a distance of h = 100 km above its surface).
Q # 3. (a) State and prove the Bernoulli's equation for an ideal fluid.
(b) Castor oil of density at room temperature is forced through a pipe of circular cross
section by a pump that maintains a gauge pressure of 950 Pa. The pipe has a diameter of 2.6 cm and a length
of 65 cm. The castor oils emerging from the free end of the pipe at atmospheric pressure is collected. After 90
s, a total of 1.23 kg has been collected. What is the coefficient of viscosity of the castor oil at this
temperature?
Q # 4. (a) What is a vector field? Explain curl of a vector field, give its physical significance and hence prove
that ⃗ ⃗ ⃗
(b) Prove that if the vector is the gradient of a scalar function then its line integral round a closed curve is
zero.
Q # 5. (a) Define center of mass of a system of many particles. Find velocity and acceleration of center of
mass and show that in the absence of external force, center of mass moves with uniform velocity.
(b) A canon whose mass M is 1300 kg fires a 72 kg ball in a horizontal direction with a speed v of 55 m/s
relative to the canon. The canon is mounted so that it can recoil freely. (i) What is the velocity of the recoiling
canon with respect to earth? (ii) What is the initial velocity of the ball with respect to the earth?
SECTION-II
Q # 6. (a) Prove that simple harmonic motion can be described as the projection of uniform circular motion
along the diameter of a circle. Find equations of displacement, velocity and acceleration of projection. Also
prove that circular motion can be regarded as combination of two identical simple harmonic motions.
(b) A pendulum is formed by pivoting a long thin rod of length L and mass m about a pivot on the rod that is
that is distance d above the center of rod, (i) find small amplitude period of this pendulum in terms of d, L, m,
and g (ii) Show that the period has a minimum value when d =0.289L.
Q # 7. (a) What is Doppler effect? Find expressions for apparent frequencies when (i) Observer is in motion
and source is at rest. (ii) Source is in motion and observer is at rest.
(b) A sound wave of intensity 1.6 µW/cm2: passes through a surface of area 4.70 cm2. How much energy
passes through the surface in one hour?
Q # 8. (a) What is plane polarization of light? Explain polarization of light by reflection and derive an
expression of Brewster law.
b) Light traveling in water with is incident on plane of a glass plaite with . At what polarizing
angle reflected light is completely polarized.
Q # 9. (a) What is diffraction grating and grating element? Derive an expression for its dispersion and
resolving power.
(b) Calculate approximately the relative intensities of the secondary maxima in the single slit Fraunhofer
diffraction pattern.
Q # 10. Write note on any two of the following:
a) Traveling waves
b) Michelson's interferometer
c) Rocket equation

105
Contact Us: aliphy2008@gmail.com, www.facebook.com/HomeOfPhysics
B.Sc. Physics Mechanics
UNIVERSITY OF SARGODHA
B.A. / B.Sc. 1st Annual Exam 2016
Physics Paper: A
Time Allowed: 3 Hours Maximum
Marks: 50
Note: Attempt any five questions in all, selecting at least two questions from each part. All questions carry
equal marks.
SECTION-I
Q # 1. (a) What are direction cosines? Prove that .
(b) Fine the Cartesian cosines of (2, -l, 2)
Q # 2. (a) What is friction? Discuss in detail static and kinetic friction. Which is greater?
(b) What is the greatest acceleration that can be generated by runner if the coefficient of static friction
between shoes and road is 0.95.
Q # 3. (a) What is rotational K.E. Derive its formula?
(b) Derive Rotational inertia of hallow cylinder.
Q # 4. (a) Define pressure of a fluid. Derive an expression for pressure gradient between the point inside a
fluid and point at its surface. Determine variation in pressure in atmosphere.
(b) A stream of water from a fexect necks down as it falls. The cross section area A0 is l.2 cm2 and that of A
is 0.35cm2. The two levels are separated by a vertical distance h= 35mm. At what rate does water flow from
the tap.
Q # 5. (a) Write short notes on any two of the following.
a) Relativity of length by the consideration of special theory of relativity.
b) Oblique collisions
c) Artificial satellite.

SECTION-II
Q # 6. (a) What is torsional oscillator? Show that its motion is SHM and find its time period
(b) A uniform disc is pivoted at its rim. Find its period for small oscillations and the length of equivalent
simple pendulum
Q # 7. (a) Compute wave speed using mechanical analysis, also calculate transverse velocity of the particle.
(b) A vibrator sets the string into motion at a frequency of 120 Hz. The string has a length of l=l.2 m and
linear density 1.2 g/m. To what value, must the tension be adjusted to give four loops?
Q # 8. (a) Explain the interference in thin film.
(b) A water film with in air is 320 nm thick. If it is illuminated with white light at normal incidence,
what color will it appear to be in reflected light?
Q # 9. (a) Discuss circular aperture diffraction.
(b) What is dispersion and resolving power?

106
Contact Us: aliphy2008@gmail.com, www.facebook.com/HomeOfPhysics
B.Sc. Physics Mechanics
UNIVERSITY OF SARGODHA
B.A. / B.Sc. 1st Annual Exam 2015
Physics Paper: A
Time Allowed: 3 Hours Maximum
Marks: 50
Note: Attempt any five questions in all, selecting at least two questions from each part. All questions carry
equal marks.
SECTION-I
Q # 1. (a) State and prove Gauss’s Divergence theorem.
(b) Prove that the magnitude of vector product of two vectors is equal to the area of parallelogram.
Q # 2. (a) What is conical pendulum? Calculate its period of revolution.
(b) A conical pendulum is formed by attaching a 53g pebble to 1.4 m string. The pebble swings around in a
circle of radius 25cm: (i) What is the period of the pebble? (ii) What is acceleration? (iii) What is the tension
in string?
Q # 3. (a) Calculate rotational inertia of solid cylinder about an axis passing through center and perpendicular
to its axis of symmetry.
(b) A disk spins on horizontal shaft mounted in bearings with angular speed of . The entire
disk and shaft assembly are placed on turn table rotating about vertical axis at counter clock
wise as we viewed it from above. Describe the rotation of disk as seen by observer in room.
Q # 4. (a) What is the equation of continuity? Prove it. What result you get from it?
(b) Crew members to escape from a damaged submarine 112 m below the surface. How much force must be
applied to pop-out hatch which is 1.22 m by 0.590 m to push it out?
Q # 5. Write short note on any two of the following:
i. Rocket Equation
ii. Mosely’s Law
iii. Psedo Forces

SECTION-II
Q # 6. (a) Derive wave equation.
(b) Calculate the speed of a transverse wave in a string of length 2.15 m and a mass 62.5 g under the tension
of 487 N.
Q # 7. (a) What are standing waves? Prove that the frequency of standing waves is quantized.
(b) Speed of a wave in a string is 172 when tension is 123 N. To what value must the tension be
increased in order to increase the wave speed to 180 .
Q # 8. (a) What is diffraction? Explain single slit diffraction in detail and hence derive diffraction equation.
(b) A slit of width “a” is illuminated by white light. For what value “a” does the first minima for red light
having wavelength of 650 nm fall θ=15°.
Q # 9. (a) hat is meant by polarization of light? How you can get a plane polarized light by polarizing sheet.
Show that in a circular polarization, the average intensity of light is proportional to the square of maximum
amplitude of light waves?
(b) We wish to see a glass plate with n =1.5 in air as a polarizer. Find the polarizing angle and the angle of
refraction.

107
Contact Us: aliphy2008@gmail.com, www.facebook.com/HomeOfPhysics
B.Sc. Physics Mechanics
UNIVERSITY OF SARGODHA
B.A. / B.Sc. 1st Annual Exam 2014
Physics Paper: A
Time Allowed: 3 Hours Maximum
Marks: 50
Note: Attempt any five questions in all, selecting at least two questions from each part. All questions carry
equal marks.
SECTION-I
Q # 1. (a) Explain Gradient of a scalar and show that for a scalar function : ⃗
(b) A particle moves along curve where t is the time, determine its velocity
and acceleration at any time.
Q # 2 (a). What is conical pendulum? Show that the period of motion oft: conical pendulum does not depend
on mass.
(b) A block is at rest on an incline plane making an angle θ with the horizontal, as the angle of inclination is
raised, it is found that slipping just begins at an angle of inclination θ =150. What is the coefficient of static
friction between block and incline?
Q # 3 (a). Define the parallel axis theorem. Find the rotational inertia of a solid rod of length L and mass M
about an axis which is perpendicular to length and passes through lite center.
(b) The angular speed of an automobile engine is increased from 1170 rev/min to 2880 rev/min in 12.6 s. Find
the angular acceleration in rev/min2. How many revolutions does the engine make during this time?
Q # 4 (a). Find the Gravitational force on a point mass when it is outside and inside a uniformly dense
spherical shell.
(b) It is desired to place a communication satellite into orbit so that it remain fixed above a given spot on the
equator of the rotating earth, what is the height above the earth of such an orbit? .
Q # 5 (a). What are postulates of special theory of relativity, write note on:
i. Time Dilation
ii. Length Contraction
(b) The mean lifetime of muons stopped in a lead block in the laboratory is measured to be 2.20 µs. The mean
life time of high speed muons in a burst of cosmic rays observed from the earth is measured to be 16.0 µs.
Find the speed of those cosmic ray muons.
Q # 6 (a). Write short note on two of the following:
i. Work Energy theorem
ii. Pseudo Forces
iii. Interference from Thin Films
SECTION-II
Q # 7. (a) Define Simple Harmonic Motion. Write down simple harmonic equation and sole it for the
displacement, time period and acceleration.
(b) In an electric shaver, the blade moves back and forth over a distance of 2 mm. The motion is simple
harmonic with frequency 120 Hz. Find (a) amplitude, (b) maximum blade speed, (c) maximum acceleration of
blade.
Q # 8. (a) Define travelling waves. Derive the expression for velocity of a wave in a string by mechanical
analysis.
(b) What are the three lowest frequencies for standing waves on a wire 9.88 m long having mass of 0.107 kg,
which is stretched under a tension of 236 N.
Q # 9. (a) What are Beats? Show analytically that number of beats per second is equal to the difference of
frequencies of two oscillators.
(b) A string of violin is a little too taut. Four beats per second are heard when it is sounded together with a
tuning fork that is vibrating actually at the pitch of concert A (440 Hz). What is the period of the violin string
vibration?
Q # 10 (a). What is meant by diffraction of light? Derive condition for maxima and minima of diffraction due
to single slit.
(b) A converging lens is of 32 mm in diameter. (a) What angular separation must two distant point objects
have to satisfy Rayleigh’s criterion? Assume λ=550 nm.

108
Contact Us: aliphy2008@gmail.com, www.facebook.com/HomeOfPhysics
B.Sc. Physics Mechanics
UNIVERSITY OF SARGODHA
B.A. / B.Sc. 1st Annual Exam 2013
Physics Paper: A
Time Allowed: 3 Hours Maximum
Marks: 50
Note: Attempt any five questions in all, selecting at least two questions from each part. All questions carry
equal marks.
SECTION-I
Q # 1. (a) State and prove Stoke’s theorem.
(b) Evaluate the following integral for close surface:

∮ ̂

Q # 2 (a). Calculate rotational inertia of a disc about an axis passing through a center and perpendicular to its
plane.
(b). Which is greater, angular momentum of earth associated with the rotation about its own axis or angular
momentum
of earth associated with its orbital motion around sun. The distance between sun and earth is .
Q # 3 (a). What is elasticity? Discuss in detail Young's Modulus and Bulk Modulus.
(b). In a hurricane air density 1.2 kg/m3 is blowing over the roof of a house at a speed of l10 km/h. (a) What is
the pressure difference between inside and outside that tends to lift the roof. (b) What should be the lifting
force on a roof of area 93 m2.
Q # 4 (a). What is Lorentz Transformation? Derive it mathematically. When Lorentz transformation reduces
to Galilean transformation.
(b) Find speed parameter β and Lorentz factor γ for a particle whose K.E. = l0 MeV if particle is electron.
Q # 5 (a). Discuss the elastic collision in two dimensions. How will you find the velocities of the two particles
after collision having mass m1 and m2. The initial velocity of the first particle is V1 and second particle of
mass m2 is at rest. The first particle scattered at an angle θ and second particle makes an angle φ along the
original direction.
(b) A gas molecule having speed of 322 ms-1 collides elastically with another molecule of the same mass
which is initially at rest. After collision the first molecule moves at an angle of 30° to its initial direction. Find
the speed of each molecule after the collision.
Q # 6. Write a short note on any two of the following:
(i) Equation of continuity of a fluid
(ii) Physical Pendulum
(iii)

SECTION-II
Q # 7 (a). Define interference of waves. Derive the formula for the condition of constructive interference.
(b) A string can vibrate with the frequency of 440 Hz and has a length of 0.34 m. What are the three longest
wavelengths of the resonance of the string?
Q # 8 (a). What is Michelson lnterferometer? Explain its construction, working and uses.
(b) In a double slit experiment with blue light of wavelength 512 nm the slits are l.2 mm apart and screen is
5.4 m from the slits. How far apart are the bright fringes as seen on the screen.
Q # 9 (a). What is tortional pendulum? Prove that its motion is SHM. Find its time period.
(b) The period of disc of radius 10.2 cm executing small oscillations about a pivot at its rim is measured to e
0.784 s. Find the value of g.
Q # 10 (a). What is diffraction grating? Explain how grating diffract light. Find its resolving power.
(b) A grating has 9600 lines uniformly spread over a width of 3 cm and is illuminated by light from mercury
vapor discharge. (i) What is expected dispersion in the third order in the vicinity of the intense green line of
wave length λ= 546nm. (ii) What is the resolving power of this grating in the fifth order?

109
Contact Us: aliphy2008@gmail.com, www.facebook.com/HomeOfPhysics
B.Sc. Physics Mechanics
UNIVERSITY OF SARGODHA
B.A. / B.Sc. 1st Annual Exam 2012
Physics Paper: A
Time Allowed: 3 Hours Maximum
Marks: 50
Note: Attempt any five questions in all, selecting at least two questions from each part. All questions carry
equal marks.
SECTION-I
Q # 1 (a). Explain Divergence of a vector field. Show that ⃗ ⃗ ⃗.
(b). If ( ) , find at point (1, -2,1).
Q # 2 (a). What is rotor? Find an expression for tangential speed of a man on the wall of the rotor.
(b). A circular curve of highway is designed for traffic moving at 60 km/h. If the radius of curve is I50 m,
what is the correct angle of banking of the road?
Q # 3 (a). Discuss inelastic collision in one dimension of two particles which stick together after collision in
Lab’s frame.
(b). Two skaters A and B mass 83kg and 55 kg respectively collide and embrace in a completely inelastic
collision. Before collision skater A was moving east with velocity 6.4 Km/h and skater B with velocity 8.8
km/h towards north. What is the velocity of couple after impact‘?
Q # 4 (a). Show that the force exerted by a uniform thin spherical sheet of mass M on a point mass 'm' when it
is outside, is the same as if all the mass of the spherical sheet were concentrated at its center.
(b) What is the gravitational potential energy of the Moon-Earth system, relative to the potential energy at
infinite separation?
Q # 5 (a). What is theory of relativity? Define its two basic postulates. Prove that , where symbols
have usual meanings.
(b) What is the momentum of a photon moving at speed of ?
Q # 6. Write a note on any two of the following:
(i) Escape velocity (integrating techniques)
(ii) Rocket Equation
(iii) Bernoulli equation
(iv) work-energy Theorem
SECTION-II
Q # 7 (a). What is physical pendulum? Write down its equation of motion and find its time period. Also find
the length of equivalent simple pendulum.
(b) Discuss briefly the Lissajous patterns.
Q # 8 (a). What is meant by interference of light? Describe the Young's double slit experiment and find the
conditions of constructive and destructive interference.
(b) The double slit arrangement is illuminated by light of wavelength , the slit are
apart, and the Screen on which the interference pattern appears is 55cm away. What is angular position of the
first minimum? What is the linear distance on screen between adjacent maximum m and m+1.
Q # 9 (a). Derive an equation of motion of travelling waves in one dimension (on a string).
(b) In an arrangement, a vibrator sets the string into motion at a frequency of 120 Hz. The string has a length
L=1.2m, and its linear mass density is 1.6 g/m. To what value must the tension be adjusted to obtain the
pattern Of motion having four loops?
Q # 10 (a). What is polarization? How the polarized light can be produced by reflection?
(b). Two polarizing sheets have their polarizing directions parallel so that the intensity Im of the transmitted
light is a maximum. Through what angle must either sheet be turned if the intensity is lo drop by one half?
Mass of moon , distance between Earth and Moon .

110
Contact Us: aliphy2008@gmail.com, www.facebook.com/HomeOfPhysics

Das könnte Ihnen auch gefallen